Download as pdf or txt
Download as pdf or txt
You are on page 1of 84

COMMON SENARIOS IN PEDIATRIC EMERGENSY OSCE STATION

1. Upper airway obstruction (Epiglotitis ,croup , F .body )


2. Infant respiratory distress – Bronchiolitis , pertussis
3. acute sever asthma - status asthmatics
4. Acute chest syndrome
5. CCF - pulmonary odema
6. Hypercyanotic spell
7. SVT ,VT ,VF
8. ARF
9. CRF emergency
10.nephrotic syndrome emergency
11.Septic shock
12.adrenal failure CAH-steroid Rx (NS)
13.anaphylactic shock
14.Neonatal sepsis
15.DKA
16.malignancy emergency –TLS
17.LOC
18.poisoning management.. ( aspirin ,TAD, iron, OPP ,paracetamol)
19.convulsion (status eplipticus ,Herpitic encephalitis)
Pediatric Basic Life Support for Healthcare

In the event of an arrest, whether respiratory or cardiac


BLS must be started ,good technique will limit the effects of hypoxia

C-A-B Sequence
The 2010 AHA Guidelines for CPR and ECC recommend a CAB sequence (chest
compressions, airway , breathing /ventilations).
BLS consists of chest compressions, airway, breathing, and defibrillation.
Because delays and interruptions in chest compressions reduce survival, the
recommended CPR begins the sequence with chest compressions before giving
rescue breaths(C-A-B rather than A-B-C).
Chest compressions can started almost immediately and require no equipment.
Positioning the head and achieving a seal for mouth-to-mouth or Bag -mask
rescue breaths take time and may require equipment that must be assembled.

High-quality CPR is the foundation of both basic and advanced life support.
For the child in cardiac arrest, a team member should perform immediate high-
quality CPR until the AED arrives.
1. Assesses victim (Steps 1 and 2, assessment and activation,10 sec of arrival):
• Checks for unresponsiveness (this MUST precede starting compressions)
• Gently stimulate the child and ask loudly, ‘Are you all right?’
• Do not shake infants, or children with suspected cervical spine injuries.
• Checks for no breathing or only gasping
If the child responds by answering or moving:
• Leave the child in the position in which you find him (not in further danger).
• Check his condition and get help if needed. Reassess him regularly.
If the child does not respond:
• Shouts for help/directs someone to call for help AND get AED/defibrillator
• Sends someone to activate emergency response
• Proceed with the resuscitation, by assessing and managing C-A-B (start Checks for pulse)

2. Checks for pulse:


• Checks brachial pulse in Infant ,carotid or femoral pulse in child
• This should take no more than 10 seconds X 60

In the absence of a pulse or < 60 /min


Delivers high-quality CPR (initiates chest compressions within 10 seconds ):
Correct placement of fingers / hand(s) in center of chest
 Infant 1 rescuer: 2 fingers just below the nipple line
 Infant 2 rescuer: 2 thumb-encircling hands just below the nipple line
 Child 1 or 2 hands on lower half of breastbone
Compression rate of at least 100/min
- 1 rescuer: 30 compressions / 2 breaths
- If two rescuer 15 compressions / 2 breaths
- at least one third the depth of the chest ( 4 – 5 cm)
- Delivers breaths that produce visible chest rise
- Compressions not interrupted until AED analyzing rhythm
- Compressions resumed immediately after shock/no shock indicated
Integrates prompt and proper use of AED with CPR:
After two minutes (2 cycle) have passed
 When the AED is available, the victim’s rhythm should be quickly assessed.
If the rhythm Shockable include ventricular fibrillation or pulseless VT.
 One shock should be given to the victim
 resumption of CPR immediately after the shock.
 CPR should continue for 2 minutes.
If the rhythm not shockable rhythm
 CPR should be resumed immediately.
 check the rhythm once more after two minutes of CPR (5 cycles ).
 Continue CPR until the victim moves or advanced life support is available.
If spontaneous heart rate and respirations have not returned, BLS is not
expediting recovery, continue CPR, but proceed to ALS
When assistance is available, the priorities are:
• Ensure that the paediatric emergency team has been called.
• Optimize oxygen intake using a bag and mask with reservoir.
• Attach monitor and determine rhythm.
• Initiate appropriate treatment protocol (APLS) with drug treatment should be
tailored to the particular dysrhythmia
 amiodarone or lidocaine for ventricular tachycardia
 adenosine for supraventricular tachycardias
 cardioversion for ventricular fibrillation—usually used in older children.

If there is a palpable pulse : Checks for no breathing or only gasping


Assess airway
Inspect: look in the mouth and nose for anything that may cause a blockage, and remove
what you can reach. Never prod blindly in the airway with a finger or with suction — you
may push the object further down the respiratory tract.
Open airway: (head-tilt/chin-lift or jaw-thrust if the cervical spine is unstable) Chin-lift is
the more important aspect of this manoeuvre. As the patient’s age increases, greater
head-tilt is required to open the airway:
• Infants— neutral position
• Child— sniffing.
Assess breathing (for 10 second) while keeping the airway open
Asses RR and effort by:
• Looking for chest and abdominal movement
• Listening at the mouth and nose for breath sounds
• Feeling for expired air movement with your cheek
If the patient is breathing effectively then
 turn child onto side(recovery position).
If the patient is not breathing effectively , require assistance ventilation.
 Give 2 rescue breaths
Rescue breathing should be done by
 mouth-to-mouth or mouth-to-nose breathing for a child
 mask over the patient's nose and mouth for a infant
 mouth-to-mask breathing
 bag-mask respirations
Assess breathing again
If successful manoeuvre ….Observation

If the chest does not rise, it most likely that the airway is not in open position.
 Reposition the Airway, and consider a jaw-thrust.
 check mouth and nose again to exclude foreign body airway obstruction.
 Remove any foreign material that you can easily reach.
 Give again 2 rescue breaths - Repeated attempt (5 times)

if all 5 attempts to make the chest rise are ineffective and failure of reposition, Foreign
body aspiration suspected mainly if :
 respiratory distress has had a sudden onset
 an unconscious, apneic infant or child then treat as foreign body airway obstruction

if after rescue breaths , chest rise, but no respiratory effort then treat as respiratory failure
basic Life Support (BLS)algorithm
Treatment the choking child (Foreign body aspiration)

Measures to open the upper airway and maintain patency may include:
Open air way head tilt-chin lift, consider a jaw-thrust if the cervical spine is unstable
check mouth and nose remove any foreign material that you can easily reach.
If patient responsiveness in distress or weak cry(partial obstruction) : encourage to cough
 If coughing effectively: encourage continue cough, observation for detoriaration
 C x ray if stable , prepear for rigid bronchoscopy (manoeuvre C/I )

If cough ineffectively(complete obstruction)


 > 1 yrs old age give abdominal thrusts (Heimlich manoeuvre) then back blows.
 < 1 yrs old age give back blows (head down in your forearm) then chest thrusts.
repeated this manoeuvre 5 times ,check if effective (open mouth) before repeated
 If successful manoeuvre proceed assessment of circulation and Observation
 If unsuccessful After 5 repeating all manoeuvre or become unresponsiveness
 activated emergency respond
 lower child to floor , try abdominal thrust on laying position if > 1 yr old age.
 proceed with BLS by CPR , delivering chest compressions 100/min
 continue CPR (15:2) - 5 cycle or for 2 min.
 Advanced life support interventions in FB aspiration will include:
• direct visualization with laryngoscope removal with Magill’s forceps or suction
• ventilator Rx with bag and mask to displace object into one bronchus
• ventilator Rx with CPAP
• attempt intubation and ventilation
• emergency needle cricothyroidotomy,while preparing rigid bronchoscopy
• emergency tracheotomy only in children over 12 yrs.
Management of respiratory arrest

•Provide breaths for the patient using bag-mask ventilation ± oropharyngeal


airway, with high concentration oxygen.

There is no necessity to intubate — adequate ventilation can be achieved


with good quality bag-mask ventilation ± oropharyngeal airway.

• Ventilate 12–20/min,aiming to make the chest rise as in a normal inspiration.


• Avoid over-inflating the chest.
• After 1 min, reassess ABC.
• Ensure that assistance is on the way.
• Proceed with resuscitation as required.

N.B. Respiratory arrest can be in conjunction with a seizure, particularly


tonic epilepsy. Stopping the seizure will facilitate ventilatory support.

If unsuccessful then intubation and invasive ventilation need


Indicators for endotracheal intubation include
• apnea
• loss of central nervous system control of respirations
• airway obstruction unrelieved by airway-opening maneuvers
• increased work of breathing that may lead to fatigue
• the need for positive EEP or a high peak inspiratory pressure
• poor airway-protective reflexes, sedation, or the need for paralysis.

Once the patient is intubated, proper tube placement is assessed by


 breath sounds
 chest rise
 instantaneous analysis of exhaled carbon dioxide (CO2) by a colorimetric device
placed within the respiratory tubing near the endotracheal tube (ETT).
 A chest x-ray then confirms the position.
Paediatric Advanced Life Support - (PALS)
When BLS is not expediting recovery, continue CPR, but proceed to ALS

1. Review ABC .
If problems arise, review ABC to assess effectiveness ,there are 8 causes of
arrest that are reversible (‘4Hs and 4Ts’) they must be treated before proceeding
• Hypoxia : oxygenation, and ventilation, reconfirm endotracheal tube placement
• Hypovolaemia : Administer fluids augment with vasopressors as necessary
• Hypothermia : initiate active internal rewarming
• Hyper/hypokalaemia, hypocalcaemia (metabolic):
• Tension pneumothorax : Needle decompression
• Toxic/therapeutic disturbance antidote , bicarbonate magnesium sulfate
• Tamponade Administer fluids; obtain bedside Echo ,Perform pericardiocentesis
• Thromboemboli : thrombolytic therapy, embolectomy

2 .Oxygenate, ventilate and chest compression:


• continue CPR
• manage airway and breathing with mask and ambu bag and high flow oxygen
• consider oropharyngeal airway
• A laryngeal mask may be used if skilled in insertion.
• insert NGT
• endotracheal intubation

If there is any deterioration in ventilation, remember DOPES.


• D isplacement of endotracheal tube.
• O bstruction of endotracheal tube or upper airway.
• P neumothorax.
• E quipment failure, e.g. oxygen not attached, not plugged in.
• S plinting of diaphragm by air in stomach—insert NGT.

ET Size ID ≥2 year = 4+ (age in yrs ÷ 4)mm


ET Length (cm) for Oral tube ≥2 years = 12 + (age in yrs ÷ 2)cm
3 .Circulation
• continue CPR
• Establish cardiac monitoring.
• Pause compressions to check rhythm and feel for pulse.
• Determine whether defibrillation is necessary
Shockable : VF and Pulseless VT.
Non-shockable (Most common arrest rhythm in pediatric age group)
• Asystole with a flat line on ECG + absence of signs of circulation
• Pulseless electrical activity(PEA)other than VF/VT does not produce a palpable pulse
• SVT (if not in shock).
• Obtain vascular access, IV or IO, so that fluids and medications can be given.
• If time permits, take bloods for bedside glucose, FBC, UEC, blood cultures ±
clotting studies, and insert second IV line.

If Vascular access lost certain drugs can be givenvia the E tube—ALAN:


• Adrenaline 100mcg/kg (0.1mL/kg of 1 in 1000 adrenaline).
• Lidocaine 2mg/kg.
• Atropine 40mcg/kg.
• Naloxone 10–100mcg/kg.

Hypoglycaemia
If fingerprick glucose is under 4mmol/L, treat with 2mL/kg of 10% dextrose.
4.Drugs
Before giving any medication, ensure that it is compatible with the means
of vascular access and the fluids running through it, e.g. amiodarone is
not compatible with 0.9% sodium chloride, so flush line before and after
administration with 5% dextrose.
During resuscitation, ensure that one member of staff records all drugs
used, the dose, route of administration, and time drug given.
Circulatory collapse
• Give fluid bolus of 20mL/kg of 0.9% sodium chloride to improve the
efficacy of chest compressions.
• If no response to the initial dose of adrenaline and there is dilated
circulation discuss the use of a vasopressor agent with senior doctor/PICU.
Persisting arrhythmia
• Consider anti-arrhythmics:
Treatment
Non-shockable : Asystole and Pulseless electrical activity
 continuous CPR 15:2 + high flow O2 via ampu bag ,intubation
 Obtain vascular access IV or IO , Give adrenaline: (repeated every 4 min)
 Repeat the cycle: (4min) and monitor rhythm every 2 min
 Give worm saline 30 ML/kg (PEA)
 Consider the use alkalising agents (Na bicarbonate).
 Consider and correct reversible causes (see above: 4Hs and 4Ts).
 bloods for bedside glucose,FBC, UEC, blood cultures ± clotting studies
 take history

Shockable: Pulseless VT and VF.


 Defibrillate synchronized DC shock 4j /kg
 Resume CPR immediately 15:2
 Get vascular access
 Repeat the cycle: Repeated CPR(2min) and defibrillation
 After 2nd shock Give adrenaline and amiodarone
 adrenaline - 10mcg/kgIV / IO or 100 mcg/kg ETT (repeated/2 min)
 amiodarone 5mg /kg IV boluse (over 30 min) (can repeated 2 times)
 Consider and correct reversible causes (4Hs and 4Ts).
 obtain bloods for glucose,FBC, UEC, blood cultures ± clotting studies
 take history

Shockable: Pulseless SVT.


 cardioversion with a synchronised DC shock (1 -2 J/kg) (can repeated 2 times)
POST-RESUSCITATION CARE

When resuscitation is successful, continuous PICU care is usually needed


to attend to the potential post ischemic multiple organ dysfunction syndromes
and the continued need for cardiac inotropic support.

Establish comprehensive monitoring so that any changes in condition


are rapidly noticed:
• continuous cardiac monitoring: pulse, BP, SpO 2;
• skin temperature ± core temperature;
• GCS.

Obtain specimens to establish baseline parameters:


• bloods—glucose, FBC, UEC, LFT, clotting studies, cross-match,ABG ± blood cultures;
• 12-lead ECG
• CXR.

Arrange transfer to PICU.


If there are no PICU facilities available in your hospital, arrange transfer by a
specialist paediatric retrieval team.
Your local PICU can give advice on management. An unstable child must not
be moved, so you are responsible for optimizing the child’s condition until the
team’s arrival

Communication:
• confer with consultants involved in the child’s care;
• notify PICU of any adverse change in the child’s condition;
• document the resuscitation in the child’s notes.

When resuscitation fails and the patient dies, attention is naturally focused
on comforting the grieving family.
Finally, someone on the team must be mindful of the legal and procedural
duties involved in a death, such as notification of the coroner, contact with the
organ/tissue transplant bank, completion of the death certificate, and
arrangements for disposition of the remains of the deceased, depending on
local regulations, family wishes, and customs.
Ventricular tachycardia VT
Ventricular tachycardia (VT) arises as a rapidly discharging ventricular
focus at a rate of 150–250 bpm caused by reentry.
These arrhythmias are usually serious and associated with symptoms of ;-
 chest pain
 palpitations
 syncope.
This rhythm may occur in ;-
 benign, monomorphic VT in infants
 myocarditis
 after a catastrophic injury
 metabolic derangement.
 digoxin or tricyclics toxicity
 torsades depointes (literally,“twisting of the points” or axis)
A type of polymorphic VT characterized by QRS amplitude that
gradually increases then decreases
 antiarrhythmic drug toxicity (procainamide) and long QT syndrome

The electrocardiogram shows;-


 regular wide-QRS complexes
 P waves that occur at a slower rate (AVdissociation).
Treatment
if Pulseless Rx as VF
if unstable Rx synchronized DC shock
if stable
 Give amiodarone 5mg/kg
 Procanamide 15mg/kg over 30 minutes.
 lidocain 1mg/kg boluse iv only use when amiodarone is unavailable
 Iv mg sulphate 50mg/kg or B blocker in Torsades de pointes
 Na bicarbonate in tricyclic poisoning
 Phenytoin in digoxin or cocaine poisoning
 Don’t give amidaron and procanmide togeather because risk of hypotension
and prolonged QT.
If respond and stabilized Maintance with long term indral for life
Supraventricular tachycardia
This is the most common childhood arrhythmia and usually in infancy
with rapid heart rate between 250 and 300 beats/min. Causes include
 febrile illness
 drug exposure (20 % )
 congenital heart disease
 Wolff–Parkinson–White (30 % ) re-entry SVT
 50 % have no known aetiology

Mechanisms of supraventricular tachycardia.


orthodromic reciprocating tachycardia
characterized by SVT due to an accessory conduction pathway between the
atria and the ventricles, seen In the reentry Wolff-Parkinson-White (WPW)
syndrome, the AC conducts from ventricle to atrium (reentry or retrograde) ;
therefore, a delta wave is not present, and the QRS is narrow, similar to other
patients with this type of SVT (orthodromic reciprocating tachycardia)

In atrioventricular nodal reentrant tachycardia (AVNRT)


A rapid heart rate due to more than one pathway in the right atrium within the
AV node and exhibit properties similar to AV nodal tissue. characterized by an
abrupt onset and cessation and tend to occur when the patient is at rest.
demonstrate a narrow QRS complex, an abnormal P-wave axis

Primary atrial tachycardia concealed ACs (e.g., atrial flutter)


originates in the atria (true SVT) and conducts via the AVN to the ventricles—in
this case, in a 2:1 ratio. In each type, the sinoatrial node (SAN) is suppressed
because of the more rapid SVT.
Clinical Manifestations
• Abrupt onset and termination.
• It can cause poor cardiac output and pulmonary oedema (heart failure) in
the neonate or young infant or fetal hydrops fetalis and intrauterine death.
• Infants—irritability, poor feeding, tachypnea.
• Older children—palpitations, chest pain, shortness of breath.
Investigations should include;-
 a 12-lead ECG
 check electrolytes (potassium, calcium)
 Echo to rule out structural heart disease - rarely (particularly Ebstein's
anomaly and cardiomyopathy, both associated with tachyarrhythmia).
The ECG will generally show
 no clear P waves regular narrow- QRS tachycardia of 250-300 beats/min .
 In the reentry SVT in Wolff–Parkinson–White (WPW) syndrome there is no
delta wave with retrograde P waves
 In the pre-excitation Wolff-Parkinson-White (WPW) syndrome, during
sinus rhythm (SR), the early antegrade activation (pre-excitation) of the
ventricle via the pathway results in a short P-R interval ,wide QRS and delta
wave (slurred initial portion of QRS complex which manifests between
episodes of SVT)
Diagnosis
• Unvarying heart rate of 240 ± 40 bpm or more.
• P waves are either abnormal or not visible
Points to remember
Sinus tachycardia can be differentiated from SVT by the following:
• Rate (>230 bpm is almost always SVT)
• Variability (heart rate in SVT does not vary)
• normal P wave axis seen in sinus tachycardia
• Condition of the patient (fever and irritability are often present in patients
with sinus tachycardia, whereas those with SVT are typically well appearing)
Management
Pt with plus and poor perfusion
 Give high flow O2
 Connect to monitor
 Evaluate rhythm by 12 ECG lead
 Give worm saline 30 ML/kg (PEA)
 Obtain vascular access IV or IO

IF PT stable.
 Vagal stimulating manoeuvres, e.g. carotid sinus massage or cold ice pack to
face, successful in about 80%.
 Intravenous adenosine is the treatment of choice, is safe and effective,
inducing atrioventricular block,with starting dose 100 mcg/kg rapid bolus
injection (max 6mg) and the ECG is observed for 10 to 15 seconds If there
recorder is no effect, give 200 then 300 mcg/kg.

IF unstable PT
 with IV access give Intravenous adenosine
 Electrical cardioversion - synchronised DC shock(1 J/kg) ,2 J/kg if not effective
IF no conversion can give :
 propranolol 0,2–0,5 mg/kg
 verampil 0.1-0.2 mg(100-200mcg)/kg IV.
 amiodarone 5mg mg/kg IV.
 Procanamide 15mg/kg over 30 minutes.
 digoxin 10 μg/kg IV as initial load 2nd dose in 6 hr, 3rd at 24 hr ,in non-WPW.

maintenance therapy
Once sinus rhythm is restored, maintenance therapy will be required.
First-line medical therapy for chronic treatment of supraventricular
tachycardia includes digoxin for non-WPW SVT, and beta-blockers eg,
propranolol 1–4 mg/kg/d , for WPW SVT.

Relapse
Even though the resting ECG may remain abnormal, 90% of children will have
no further attacks after infancy,treatment is therefore stopped at 1 year of
age,Those who relapse thereafter are usually treated with percutaneous
radiofrequency ablation or cryoablation of the accessory pathway.
.
Bradycardia
if PR = 2Ss-4Ss , 0.08 -0.16 and P related to QRS with normal QRS… This Sinus bradycardia
Causes;
athletic ,Hypoxia, hypothermia, hypokalemia , increased ICP, digoxin, B-blockers, CC blockers
Rx ; give O2 and check BP,
 if nl BP ,determine causes and observation
 if low Atropin 0.2mg/kg , adrenaline 0.2mcg/kg ,isoproternol 0.1mcg/kg/min, pacing
 if high With increase ICP give manitol ,
 if high without increase ICP give hydralazine , labetolol ,nitroprusside
 if nl BP ,determine causes and observation
PEARS Systematic Approach to the Seriously III
or Injured Child
Begin Initial Impression
During the initial impression, look and listen to gather information about
• Consciousness , Breathing and Color
Identify a Life-Threatening Condition
Signs of a life-threatening condition may include the following:
 Complete or severe airway obstruction
 Apnea, slow respiratory rate, very fast respiratory rate
 very increased or inadequate respiratory effort
 Absence of palpable pulses, poor perfusion, hypotension, bradycardia
 Unresponsiveness, decreased level of consciousness
 Significant hypothermia, significant bleeding, petechiae or purpura
This need to start life support interventions and Get help

If not life threatening


This need to continue with the systematic approach
Evaluate ,Identify problem ,Intervene and Ongoing Sequence
Evaluate
primary assessment : ABCDE ,vital signs and pulse oximetry
Secondary assessment:A focused medical history and physical examination
Diagnostic tests : Laboratory, radiographic, and other advanced tests
Identify respiratory circulatory problem
On the basis of your evaluation, try to identify the type and severity of the
child’s clinical condition.
Respiratory problem and severity (mild or severe respiratory distress)
• Upper airway obstruction
• Lower airway obstruction
• Lung tissue disease
• Disordered control of breathing
• Mild respiratory distress
• Severe respiratory distress
Circulatory problem and severity
• Hypovolemic shock
• Distributive shock
• Cardiogenic shock
• Obstructive shock
Intervene
On the basis of your identification of the child’s clinical condition, intervene
with appropriate actions. Your actions will be determined by your scope of
practice and local protocol.
Interventions for PEARS providers may include
• Getting help by activating a medical emergency or rapid response team
• Starting CPR
• Obtaining an AED, code cart, or monitor/defibrillator
• Attaching a heart monitor and a pulse oximeter
• Positioning the child
• Giving oxygen
• Providing nebulizer therapy or using an epinephrine autoinjector
• Giving a fluid bolus

Ongoing Sequence
The sequence of evaluate-identify-intervene is ongoing until the child is stable.
Remember to repeat the evaluate-identify-intervene sequence after each
intervention and when the child’s condition changes or deteriorates
Primary Assessment Overview
the primary assessment is a hands-on evaluation. Here you use an ABCDE
approach to assess
• Airway
• Breathing
• Circulation
• Disability
• Exposure
Airway and Breathing Assessment and Identification of Respiratory Problems”
Circulation, Disability, and Exposure: Assessment and Identification of Shock”
Airway and Breathing
Evaluate airway
Assess the airway to determine whether it is open and clear or obstructed .:
• Look for movement of the chest or abdomen
• Listen for bilateral breath sounds and air movement
• Listening at the mouth and nose for breath sounds
The obstruction may be in the upper airway with .
• Increased inspiratory effort with retractions
• Abnormal inspiratory sounds (snoring or high-pitched stridor)
• Decreased air movement despite increased respiratory effort
Assess breathing (for 10 second X 6)
Abnormal respirations include Rate and Pattern
 Fast RR (tachypnea) Slow RR and Apnea
Evaluate respiratory effort
• Nasal flaring /Retractions
• Retractions with stridor or an inspiratory snoring sound suggest UAO
• Retractions with expiratory wheezing suggest BA,bronchiolitis
• Retractions with grunting or labored breathing suggest lung tissue disease
•Apnea ,Weak cry or cough inadequate respiratory effort
Assess chest expansion and air movement with a stethoscope
Lung and Airway Sounds
 Snoring /Stridor/Barking Cough /Hoarseness /Grunting /Wheez /Crackles
Oxygen Saturation by A pulse oximeter
Identification of Respiratory Problems and its severity

Decreased Oxygenation (Hypoxemia) pneumonia ,pulmonary edema ,asthma


Decreased Ventilation (Hypercarbia) inadequate effort, Airway obstruction
Upper or lower airway obstruction: croup, epiglottitis, BA, bronchiolitis
Lung tissue disease : pneumonia
Disordered control of breathing: toxin, CNS trauma
Mild or Severe Respiratory Problems

Upper Airway Obstruction


• Croup
• Anaphylaxis
• aspiration of foreign body
• infection
• narrowing of the airway below the vocal cords in endotracheal intubation.
Signs of Upper Airway Obstruction
• Increased respiratory rate and effort (eg, retractions, nasal flaring)
• Decreased air movement
• Stridor (typically inspiratory)
• Barking cough
• Snoring or gurgling
• Hoarseness
Stridor DDx: croup, epiglottitis, bacterial tracheitis , laryngomalecia, FB.
Lower Airway Obstruction
• Asthma and Bronchiolitis
Signs of Low er Airway Obstruction
• Increased respiratory rate and effort (eg, retractions, nasal flaring)
• Decreased air movement
• Prolonged expiration
• Wheezing
Lung Tissue Disease
• Pneumonia: bacterial, viral, fungal, chemical
• congestive heart failure or leaking of fluids into the tissues (eg, sepsis, ARDS)
• Trauma (eg, lung bruise)
• Allergic reactions /Toxin

Signs of Lung Tissue Disease


• Increased respiratory rate (often marked)
• Increased respiratory effort (especially during inspiration)
• Decreased air movement
• Grunting
• Crackles

Disordered Control of Breathing


Common causes of disordered control of breathing are
• Increased ICP (cerebral edema) ,trauma, tumor, infection, hydrocephalus
• Seizures
• Poisoning or drug overdose
• Neuromuscular disease
The following are signs of disordered control of breathing:
• Irregular respiratory pattern
• Inadequate or irregular respiratory
• Normal or decreased air movement
• Possible signs of upper airway obstruction
Management of Respiratory Problems
Acute upper airways obstruction
Differential diagnosis
•Croup Viral laryngotracheitis (very common)
•Acute-on-chronic stridor, e.g. from a floppy larynx (laryngomalacia)
•Bacterial tracheitis (rare)
•Rare causes Epiglottitis
•Inhalation of smoke and hot air in fires
•Trauma to the throat
•Retropharyngeal abscess
•Laryngeal foreign body
•Allergic laryngeal oedema (angioedema)
•Tetany due to poor vitamin D intake
•Infectious mononucleosis
•Diphtheria
•measels
General Management of Upper Airway Obstruction
initial interventions ( ABC)
relieving the obstruction include the following:
• Reduce anxiety by staff being calm, confident and well organised.
• Avoid unnecessary agitation of the child, (Don't examine the throat!)
• Position the child.
- Allow the child to assume a position of comfort.
- If a decreased level of consciousness, turn the child on his side to open airway.
• Remove any object that you see obstructing the airway.
• Suction the nose, mouth, or both.
• Observe carefully for signs of hypoxia or deterioration.
• Reduce airway swelling by using drugs, such as nebulized epinephrine.
• If you recognize signs of severe upper airway obstruction, get help.
• A provider with skill and experience in airway management may be needed
to establish an airway. Failure to aggressively treat a severe upper airway
obstruction may lead to respiratory and cardiac arrest.
• If respiratory failure develops from increasing airways obstruction,
exhaustion or secretions blocking the airway, urgent tracheal intubation is
required.
• When severe upper airway obstruction is not present, infants and children
may benefit from the use of airway adjuncts, such as an OPA.
Laryngeal and tracheal infections
Croup
Viral croup accounts for over 95% of laryngotracheal infections.
Parainfluenza viruses are the commonest, but other viruses, such as
metapneumovirus, RSV and influenza, can produce a similar clinical picture.
Croup occurs from 6 months to 6 years of age
peak incidence is in the second year of life.
commonest in the autumn.
The typical features are of a barking cough, harsh stridor and hoarseness
usually preceded by fever and coryza.
The symptoms often start, and are worse, at night.
Management

When the upper airway obstruction is mild


 Inhalation of warm moist air
 Consider Oral dexamethasone 0.15mg/kg, Prednisolone 2mg/kg,single dose
 Minimal interference
 Continue oral feeds
The parents need to observe the child closely for the signs worsining.

Moderat upper airway obstruction


 NPO / IV fluids
 Oxygen if SPO2 < 92% (measured by pulse oximeter)
 Prednisolone 2mg/kg /oral , Dexa 0.3mg/kg / IV
 Inhalation of nebulized adrenalin 1: 1000–0.5 ml in 3 ml NS, (max. 3times)
 Budesonide nebulization(2mg)/4ml.NS or 400mcg via inhaler/spacer (single)
 if improve ,stridor settles ,no cough,no RD send home with 4 hrs with
 Dexa 0.3mg/kg ,Or Prednisolone 2mg/kg, single oral daily dos(for 72 hrs)
 if improve but remain mild stridor and mild RD but no cyanosis
 admit to word for close observation ,Dexa 1 mg/kg/day IV tid (72 hrs)
 if not improve or worsning treated as sever croup

in severe upper airways obstruction


 NPO / IV fluids
 Oxygen if SPO2 < 92% (measured by pulse oximeter)
 nebulized adrenalin 1: 1000 0.5 ml in 3 ml NS, can be repeated (max. 3times)
 Steroids (mainly if adrenaline C/I as in TOF).
–Budesonide nebulization2ml(2mg)/4ml.NS or 400mcg inhaler/spacer (single)
–Dexa 0.3mg/kg / IV
 If not improve Repeat adrenaline/ 3times.
 Discuss with senior colleague.
 Arrange ICU admitin
 Call senior available anaesthetis
 Forewarn anaesthetist and intensive care.
Acute epiglottitis
Acute epiglottitis is a life-threatening emergency due to respiratory obstruction.
It is caused by H. influenzae type b.
Epiglottitis is most common in children aged 1-6 years but affects all age.
DDx
 Angioneuropathic edema of supraglottic structures
 Anaphylaxis
 C1-esterase deficiency
 Caustic ingestion
 Thermal burns of epiglottis
 infectious mononucleosis
 Laryngotracheitis
 Blunt Trauma
Treatment
If the Dx of epiglottitis is suspected, urgent hospital admission and Rx.
Initial assesment to distinguish between epiglottitis and croup
 The onset of epiglottitis is often very acute with: high fever
 ill,toxic-looking, sits immobile, upright, with an open mouth
 unable to speaking or swallowing; saliva drools down the chin
 In contrast to harsh braking cough in croup,cough is minimal or absent.
 soft inspiratory stridor and cough is minimal or absent.
 rapidly increasing respiratory difficulty over hours
 In contrast to hoars voice in viral croup, voice is muffled.
note
 Lets baby in sitting position on mother lap
 Don’t examine the throat with a spatula
 Don’t try to drawal blood for investigation or send him for x ray
initiated treatment without delay.
 ABC /NPO and Moist O2 inhalation without agitation
 A senior anaesthetist and ENT surgeon should be summoned
 The child should be transferred directly to the ICU or an anaesthetic room.
 intubated under controlled conditions with a general anaesthetic.
 If this impossible and urgent tracheostomy is life-saving,consult anesthesia
Only after the airway is secured
 blood be taken for culture (high WBC and neutrophils and band forms)
 A culture of the epiglottis is usually positive for HIB but the result may not
be available until the child is ready for discharge.
 Start IV fluid and AB such as cefuroxime ,Ceftriaxone ,cefotaxime, Unasyn ,
Timentin, Zosyn, levaquin, gatifloxacin.
Amoxicillin should not be used due to noted resistance..

X-Ray Lateral neck-

enlarged hypopharynx and forward neck


extension with “thumbprinting” of epiglottitis

The tracheal tube usually be removed after 24 hours and AB given for 3-5 days.
With appropriate treatment,most children recover completely within 2-3 days.
Prevention
HiB vaccination !!!
Family Members, day-care workers, health-care workers exposed:
Give Rifampin Ppx. of 300 mg q12h x 2d
Allergy Reaction and Anaphylactic shock
Differential diagnosis
• life-threatening asthma ,septic shock ,Faint (vasovagal episode) ,Panic attack.
• Breath-holding episode in child ,Idiopathic (non-allergic) urticaria or angioedema.
Initial assessment of severity
You must be able to identify if an allergic reaction is mild or severe (anaphylaxis).
Signs of a mild allergic reaction are
• Stuffy nose, sneezing, and itching around the eyes
• Itching of the skin or mucous membranes raised, red rash on the skin (hives)
The signs of a severe allergic reaction (Anaphylaxis) = allergy + cardiorespiratory comprise
• urticarial ,itching ,swelling of the lips, tongue, and face, vomting ,diarrhea
• Trouble breathing, cough ,stridor ,wheezing ,cyanosis
• Signs of shock
Interventions for Allergic Reaction
If pt has skin allergy only
 See medicAlert bracelet or necklace,Look for any allergen and remove
 Ask the child or caregiver about any history of allergy or anaphylaxis.
 reassurance + oral antihistamine
If pt has skin allergy and wheezing
 ABC , give O2 + Ventolin , albuterol by MDI or nebulizer solution ,consider adrenaline
 observation for 6 hrs reassess for progression
 give oral antihistamine+/- oral prednisolone
If pt has skin allergy+cardiorespiratory comprise(UAO + hypotension)Anaphylactic shock
 ABC - Give high flow O2 ,Lie pt down and rise legs
 Give IM adrenaline (1:1000 ,10 mcg/kg / 0.3ml <6yrs , 0.5ml > 6 yrs ,repeated 5 min) .
 Consider adrenaline nebulizer
 Give NS or LR - 20 mL/kg IV bolus. repeat as needed.
See respond and monitor ECG , BP, SaO2
if respond : observation for 6 hrs reassess, oral antihistamine +/- oral prednisolone
if not respond + R distress, risk further airway swelling prepare for EI
 Call pediatric senior ,ENT specialist ICU or anesthetist
 Repeated IM adrenaline ,give adrenaline infusion in PICU after 3 min if no respond
 Ongoing upper airway obstruction : Call ENT specialist and repeated IM adrenaline
 Ongoing shock : Repeat NS / LR 20 mL/kg IV bolus ,Consider EI and further adrenalin
 Ongoing lower airway obstruction
 consider Ventolin , albuterol by MDI or nebulizer solution
 consider iv hydrocortisone: 50 mg <6yrs , - 100mg > 6 yrs
 consider iv aminophylline 5mg/kg bolus, then 1 mg/kg/hr
 H 1 blocker consider chlorphenamine(Piriton) 0.2mg/kg(2,5 <6yrs ,5mg > 6 yrs)
 H 1 blocker Diphenhydramin(Benadryl) 1mg/kg iv
 H 2 blocker rantidine 1 mg/kg diluted in D5 over 5 min.
Discharge from hospital

Patients who have had a suspected anaphylactic reaction (i.e. an airway, breathing or
circulation problem) should be treated and then observed for at least 6 hours in a clinical
area with facilities for treating life-threatening ABC problems.
They should then be reviewed by a senior clinician and a decision made about the need
for further treatment or a longer period of observation.
Patients with a good response to initial treatment should be warned of the possibility of
an early recurrence of symptoms and in some circumstances should be kept under
observation for up to 24 hours.

This caution is particularly applicable to:


• Severe reactions with slow onset caused by idiopathic anaphylaxis.
• Reactions in individuals with severe asthma .
• Reactions with the possibility of continuing absorption of allergen.
• Patients with a previous history of biphasic reactions (twice attack).
• Patients presenting in the evening or at night
• Patients in areas where access to emergency care is difficult.

Before discharge from hospital all patients must be:


• Reviewed by a senior clinician.
• Given clear instructions to return to hospital if symptoms return.
• Considered for anti-histamines and oral steroid therapy for up to 3 days.
This is helpful for treatment of urticaria and may decrease the chance of further
reaction
Foreign-Body Aspiration
History of Upper airway obstruction
 History of choking (force feed baby, vomiting ,GERD) (Toddlers mobile – children)
 Sudden Cough ,Stridor , RD and cyanosis ,unresponsive
history of lower airway obstruction
 cough
 Recurrent or persistent pneumonia not responding to treatment
 Sudden onset unilateral wheeze
 Persistent wheeze unresponsive to Ventolin
 Persistent cough with no explanation
 Hemoptysis

Differential diagnosis
Upper airway obstruction
 Croup Viral laryngotracheitis (very common)
 Bacterial tracheitis (rare)
 Rare causes Epiglottitis
 Inhalation of smoke and hot air in fires
 Trauma to the throat
 Retropharyngeal abscess
 Allergic laryngeal oedema (angioedema)
 Infectious mononucleosis
 Diphtheria

lower airway obstruction


• URTI
• Pneumonia
• Asthma
• Acute bronchitis
Assessment and stabilization of pt suspected UAO
check mouth and nose remove any foreign material that you can easily reach.
Open air way head tilt-chin lift, consider a jaw-thrust if the cervical spine is unstable
If child in RD ,unable to speak , cry , cough ineffectively this complete UAO(fatal)
 If > 1 yrs old age, give abdominal thrusts (Heimlich manoeuvre) then back blows.
 If < 1 yrs old age ,give chest thrusts then back blows
 repeated this manoeuvre 5 times ,check if effective (open mouth) before repeated
 If successful manoeuvre proceed assessment of circulation and Observation
 If unsuccessful After 5 repeating all manoeuvre or become unresponsiveness
 proceed to CPR ,and ALS interventions in Foreign body aspiration will include:
 ventilatory Rx with bag and mask to displace object into one bronchus
 ventilatory Rx with CPAP
 attempt intubation and ventilation
 emergency needle cricothyroidotomy,while preparing urgent rigid bronchoscopy
 emergency tracheotomy only in children over 12 yrs.
 direct visualization with laryngoscope removal with Magill’s forceps or suction;
If child in RD , speak , cry , effectively cough this incomplete UAO
 manoeuvre contraindicated because risk of dislodge FB deeper ,cause complete UAO
 encourage continue cough ,observation for deterioration
 ensure that assistance has been called ,call 911
 transfer to specialist centre for direct visualization with rigid laryngoscope removal
 if pt stable enough ,do chest x ray PA and lateral view
Assessment and stabilization of pt suspected LAO
Physical exam
 Asymmetric chest signs
 Unilateral absence of breath sounds
 Persistent atelectasis
 Localized wheeze
Management of Lower Airway problem
 Check ABC and stabilized pt with Support adequate oxygenation and NPO
 Provide nebulized albuterol
 Iv hydrocortisone 5mg /kg
 Imaging CXR (ask for inspiration/expiration films or RT and LT lat. decubitus)
 See Radio-opaque objects
 FB usually enter RT lung(RT bronchus –more continuous with trachea)
 FB may also be in esophagus and compress trachea (look at lateral film)
 See air-trapping on expiration(ball-valve)unilateral hyperinflation(partial)
 Trachea shifts to unaffected side
 See segmental collapse ,lobar consolidation,broncheoctasis (complete)
 transfer to specialist centre for direct visualization with rigid laryngoscope
Bronchiolitis
Bronchiolitis is a seasonal viral respiratory illness that most commonly presents in
infants between 3 and 6 months of age. It occurs in winter ,main causative agent is
respiratory syncytial virus (RSV) in around 75% of cases (serotypes A and B).
Clinical examination finiding
 Signs of respiratory distress – mainly tachypnoea, recession
 Cough – often has a characteristic dry, wheezy quality
 Nasal discharge
 Hyperexpanded, barrel-shaped chest
 Fine inspiratory crackles
 Expiratory wheeze
 Air entry may be unequal due to consolidation caused by atelectasis (around 30%)

Differential diagnoses include


 CHD: ventricular septal defect/atrioventricular septal defect, myocarditis,TAPVR
 Cystic fibrosis
 Bronchopneumonia: staphylococcal , pneumococcal , aspiration, pertussis.
 Post-viral wheeze
 Foreign body
Management
Mainstay is supportive care with frequent monitoring
Supplementary humidified O2 by headbox or nasal cannulae; mandatory if Pso2< 93%.
Additional respiratory support by CPAP or transfer to a PICU if ventilation is required.
Hydration and providing help with feeding. Nasogastric feeds (small, frequent amounts)
Intravenous fluids may be required, bearing in mind that many patients develop SIADH
Suction and saline nose drops can be useful for secretions
Pharmacotherapy
The vast majority of patients do not benefit from any drugs and this should be stated to the
examiners if you are asked.
Ipratropium bromide via nebuliser reverses some of the short-term effects of bronchiolitis
but has not been shown to alter mortality or length of hospitalisation.
Nebulised adrenaline but it is currently not recommended.
Montelukast has been tried, although there is insufficient evidence to its routine use.
Nebulised ribavirin ,reduce viral shedding and shorten length of hospital stay in one study.
Steroids are of no benefit during the acute phase. some use in obliterative bronchiolitis.
This is a destructive form of bronchiolitis, usually associated with adenovirus, which
becomes chronic and results in bronchiectasis.
Antibiotics if there is clinical suspicion of super- bacterial infection –5% of patients.
Palivizumab, an anti-RSV humanised monoclonal antibody, is available for prophylaxis
against severe illness in high-risk patients (CHD,CF ,CLD,Immunodeficiency) , it is very
expensive
Acute asthma
Witch each acute attack, the duration of symptoms, the treatment already given and the
course of previous attacks should be noted.

Asthma Exacerbations and their management.


Asthma exacerbations are acute or subacute episodes of progressively worsening
symptoms and airflow obstruction.
Airflow obstruction during exacerbations can become extensive, resulting in life
threatening respiratory insufficiency.

Severe, progressive asthma exacerbations need to be managed in a medical setting,


with administration of supplemental oxygen as first-line therapy and close monitoring
for potential worsening.
Complications that can occur during severe exacerbations include
 atelectasis
 Pneumomediastinum
 pneumothorax

A severe exacerbation of asthma that does not improve with standard therapy is
termed status asthmaticus.

Immediate management of an asthma exacerbation involves


 a rapid evaluation of the severity of obstruction
 assessment of risk for further clinical deterioration .

For most patients, exacerbations will improve with frequent bronchodilator treatments
and a systemic corticosteroid course.
The optimal management of a child with an asthma exacerbation should include,
however, a more comprehensive assessment of the events leading up to the
exacerbation and the underlying disease severity.
Indeed, the frequency and severity of asthma exacerbations helps to define the severity
of a patient's asthma.
Whereas most children who experience life-threatening asthma episodes have
moderate to severe asthma by other criteria, some children with asthma appear to have
mild disease except when they suffer severe, even near-fatal exacerbations.
Management of mild Asthma Exacerbations.
All children with asthma should have a written action plan to guide their recognition
and management of exacerbations, along with the necessary medications and tools to
manage them.

immediate Rx with rescue medication (inhaled SABA ,up to 3 puffs in 1 hr).


A good response is seen by resolution of symptoms within 1 hr, no further symptoms over
next 4 hr, improvement in PEF to at least 80% of personal best.
The child's physician should be contacted for follow-up, especially if bronchodilators are
required repeatedly over the next 24–48 hr.

If the child has an incomplete response to initial treatment with rescue medication
(persistent symptoms and/or a PEF <80% of personal best), a short course of oral
corticosteroid therapy (prednisone 1–2 mg/kg/day [not to exceed 60 mg/day] for 4 days)
in addition to inhaled β-agonist therapy should be instituted.
The physician should also be contacted for further instructions. Immediate medical
attention should be sought for
 severe exacerbations
 persistent signs of respiratory distress
 lack of expected response or sustained improvement after initial Rx
 further deterioration
 high-risk factors for asthma morbidity or mortality (history of severe exacerbations).

For patients with severe asthma and/or a history of life-threatening episodes, especially
if abrupt-onset in nature, providing an injectable form of epinephrine (EpiPen) and
possibly portable oxygen at home should be considered.
Use of either of these extreme measures for home management of asthma
exacerbations would be an indication to call 911 for emergency support services.
General management
1.Assessment
• Physical examination findings:
 vital signs RR,HR,Tem.
 breathlessness
 air movement
 use of accessory muscles
 retractions
 anxiety level
 alteration in mental status
• Pulse oximetry
• the peak expiratory flow rate should be routine in school-age children
• blood gases are only indicated in life threatening or refractory cases.
• Lung function (in mod to severe distress or history of labile disease)
• A chest X-ray if asymmetry of chest ( pneumothorax, collapse)or signs of severe infection.
2.Classified Severity
Moderate
• Oxygen saturation > 92%
• Peak flow > 50% predicted or best value(age over 5 years)
• No clinical features of severe asthma
Severe
• Too breathless to talk or feed (better guide to severity).
• Use of accessory neck muscles (better guide to severity).
• Oxygen saturation < 92% (better guide to severity).
• Peak flow < 50% predicted or best value(age over 5 years)
• Respirations (poor guide to severity).
 50/min (age2–5 years)
 30/min (age over 5 years)
• Pulse (better guide to severity).
 130/min (age 2–5 years)
 120/min (age over 5 years)
Life threatening
• Silent chest
• Poor respiratory effort
• Altered consciousness
• Cyanosis
• Oxygen saturation < 92%
• Peak flow < 33% predicted or best value(age over 5 years)
3.Risk assessment of acute severe asthma, morbidity and mortality
• Onset of current exacerbation ,Frequency and severity of daytime and nighttime
• Frequency of rescue bronchodilator use
• Current medications allergies , and potential triggers smoke exposure ,Air pollution
• History of systemic steroid, ER visits,hospitalization,intubation, or life-threatening attack
 Previous severe rapidly obstruction asthma exacerbation
 Severe airways hyperresponsiveness (AHR)
 Increasing and large diurnal variation in peak flows
 Poor response to systemic corticosteroid therapy
 Male gender
 Low birthweight
 Non-white (especially black) ethnicity
 Poverty
 Crowding
 Mother <20 yr old
 Mother with less than high school education
 Inadequate medical care
 No regular medical care (only emergent)
4.treatment
Interventions are based on clinical severity on arrival, response to initial therapy, and
presence of risk factors that are associated with asthma morbidity and mortality.

For moderate asthma with


No clinical features of severe asthma ,Oxygen saturation >92% ,Peak flow > 50%
Give SABA via (MDI) (if >5yrs ) with spacer and facial mask (if < 5yrs )
2-4puffs every 20 min for 3 doses Ventolin (100μg) ,terbutilin (50μg) or Albuterol (90μg)
Increase 2puffs every 2min up to 10puffs every 20min according respond
If poor respond consider oral prednisolone : 20 mg age 2- 5 years / 40 mg age >5 years
Reassess respond to Rx within 1hr HR ,RR,Pso2 , PEF (respond ,no respond or worsing)
If good respond continue SABA every1 hrs as required and see hourly
• discharge within 4 hrs if has Criteria for discharge
 No risk of low threshold for admission, severity or death
 sustained improvement in symptoms for 4 hr
 normal physical findings
 PEF >70% of predicted or personal best (if >5yrs)
 an oxygen saturation >92% on room air for 4 hr.
If no respond , low threshold for admission, severity or death ,Arranged for admission
• Criteria for admission to hospital
Children require hospital admission if, after high-dose inhaled bronchodilator
therapy, they have not adequately responded ,there is :
 persisting breathlessness
 tachypnoea
 pt exhausted
 have a marked reduction in their predicted (or usual) peak flow rate
 have a reduced oxygen saturation (<92% in air).

For severe exacerbations with


Clinical features of severe asthma ,Oxygen saturation < 92% Peak flow < 50%
Give oxygen + SABA via (MDI >5yrs ),(spacer and mask < 5yrs)
10 puffs every 20 min for 3 doses Ventolin (100μg) ,terbutilin (50μg) or Albuterol (90μg)
Give oral prednisolone or IV steroid (hydrocortisone) 50mg/kg(2-5 yrs)100mg/kg( >5 yrs)
if poor respond nebulised ipratropium 0.5 mg ,repeated bronchodilator every 20min.
Reassess respond to Rx within 1hr HR ,RR,Pso2 , PEF (respond ,no respond or worsing)
If good respond continue SABA every 1 hr as required and see hourly for 4 hrs
• discharge within 4 hrs if no risk of low threshold for admission, severity or death
If no respond , low threshold for admission, severity or death Arranged for admission
For life threating exacerbations with
Clinical features of severe asthma ,Oxygen saturation < 92% Peak flow < 33%
Give continue oxygen via mask
Give nebulizer β2-bronchodilator with Ventolin 0.2ml/kg in 2ml NS or Albuterol 0.15 mg/kg
• Every 20min ,3doses,then every 1–4hr,or 0.5 mg/kg/hr by continuous nebulizer.
Give nebulised ipratropium 0.5 mg q6hr
Give IV steroid (hydrocortisone) 50mg/kg(2-5 yrs)100mg/kg( >5 yrs)
Discussed with senior clinician, PICU or paediatrician
Reassess respond to Rx within 1hr HR ,RR,Pso2 , PEF (respond ,no or persist life threating)
If good respond continue SABA nebulizer every 1 -hr as required and see hourly for 4 hrs
• discharge within 4 hrs if no risk of low threshold for admission, severity or death
If no respond Arranged immediate transfer to HDU/PICU and consider
 Chest x ray ,ABG ,CBC , serum electrolytes
 IV Ventolin ,a loading dose 15 mcg/kg over 20 min, followed by 1-5 mcg/kg /min
 IV Aminophylline ,a loading dose 5mg/kg over 20 minutes, followed by 1mg/kg/hr.
 IV Mg sulphate 40 mg/kg bolos (child > 5 yrs).
 IM epinephrine ,0,1mg/kg(1:1000) also can be given
 Antibiotics are only given if there are clinical features of bacterial infection.
 Administration of fluids at or slightly below maintenance fluid requirements is
recommended this situation is the association of increased SIADH.
 Despite intensive therapy, some asthmatic children will remain critically ill and at risk
for respiratory failure, intubation, and mechanical ventilation may required.
 Elective tracheal intubation with rapid-induction sedatives and paralytic agents is
safer than emergency intubation.
 Mechanical ventilation aims to achieve adequate oxygenation while tolerating mild to
moderate hypercapnia (Pco2 50–70 mm Hg) to minimize barotrauma.
 Volume-cycled ventilators, using starting parameters that can achieve these goals.
• short inspiratory and long expiratory times
• 10–15 mL/kg tidal volume
• 8–15 breaths/min
• peak pressures <60 cm H2O
• without positive end-expiratory pressure

Discharge medications include administration of


 an inhaled β-agonist up to every 3–4 hr
 plus a 3–7 day course of an oral corticosteroid.
The patient may be discharged to home with
 come back if recurrence
 a written individualised asthma action plan.
 inhaler technique should be reviewed and altered if inadequate.
 Follow-up arrangements and monitor by the general practitioner or, for the more
problematic patients, by a paediatrician
Management of Infants and Young Children.
Moderate exacerbations with
• able to feed ,use of accessory neck muscles ,audible wheezing
• Oxygen saturation > 92%
Sever exacerbations with
• unable to feed ,marked RD ,cyanosis
• Oxygen saturation < 92%
For life threating exacerbations with
• Apnea , bradycardia ,poor respiratory effort
• Oxygen saturation < 92%
Treatment
Give oxygen via facial mask or nasal prong
β2-bronchodilator via spacer and face mask or nebulizer and repeated every 1-4hr with.
• Ventolin ,terbutilin 10 puffs by MDI or nebulizer 1% 0.2ml/kg in 2ml NS for 10 min
Closed monitor RR ,HR ,Sao2
Hydration and nursing care
Consider need for chest x ray
If poor respond
 Added nebulised ipratropium 0.5 mg q6–8 hr (tid-qid) as needed
 Consider oral soluble prednisolone tab.10 mg
If no respond or any signs of life threating features
 Discussed with senior paediatrician or PICU team
 Transfer to PICU
If good respond continue SABA nebulizer every1 hr as required and see hourly for 4 hrs
• discharge within 4 hrs if has Criteria for discharge
 No risk of low threshold for admission, severity or death
 sustained improvement in symptoms for 4 hr
 normal physical findings
 an oxygen saturation >92% on room air for 4 hr.

Discharge medications include administration of


 an inhaled β-agonist up to every 3–4 hr
 plus a 3 days course of an oral corticosteroid.

The patient may be discharged to home with


• come back if recurrence
• a written individualised asthma action plan.
• inhaler technique should be reviewed and altered if inadequate.
• Follow-up arrangements and monitor by the general practitioner or, for the more
problematic patients, by a paediatrician
Management of Lung Tissue Disease
General Management of Lung Tissue Disease

In addition to the general interventions for respiratory distress (ABC)

Monitor clinical signs of circulation.

Support as necessary.

If the child has lung tissue disease due to infectious pneumonia, consider the
following interventions:

• Give oxygen.
• Draw blood cultures and administer antibiotics
• If wheezing is present, administer albuterol by MDI (with spacer) or
nebulizer solution.
• If the child has fever, take measures to reduce it.

*lt may not be necessary to draw blood cultures before antibiotics are given.
Follow facility protocol.
Management of cyanotic spells
Give O2 100%

bolus fluid to increase SVR

knee/chest position to increase SVR

NaHco3 for acidosis

Morphine iv(0.3-1mg/kg/day/q4h)

indral iv (1mg/kg/day/q6h)

diuretics are contraindicated

Intractable spells need intubation, paralysis, and ventilation ,emergency BTS


Acute chest syndrome
Acute chest syndrome (ACS) is the most common cause of death and the
second most common cause of hospitalization in SCD pt >5 yrs old age

It is defined as the development of a new pulmonary infiltrate accompanied


by symptoms including fever, chest pain, tachypnea, cough, hypoxemia and
wheezing ,Acute chest syndrome is caused by infection, infarction and/or fat
embolization , 50% of ACS events are associated with infections, including
viruses, atypical bacteria including Mycoplasma and Chlamydia and less
frequently with Streptococcus pneumoniae.
Parvovirus B19 infection can also result in ACS not only by direct infection,
but also by causing bone marrow necrosis with subsequent pulmonary fat
embolism (PFE) syndrome and they can be life-threatening if large amounts
are released to the lungs.
In about half of the cases, ACS develops during hospitalization (often for
vaso-occlusive) where fat embolization and hypoventilation contribute to the
pathophysiology

Laboratory findings:
• White blood cell count is often elevated
• Hemoglobin level often falls to below baseline values
• Thrombocytosis may be present and often follows an episode of ACS
• Secretory phospholipase 2 (an inflammatory mediator) are elevated .
The combination of fever and elevated phospholipase 2 levels predicts a
high risk of developing ACS in patients hospitalized with vaso-occlusive pain.
Management
Evaluations:
Chest radiograph
Radiographs may show single-lobe involvement, most often the left lower lobe, and
when multiple lobes are involved, usually both lower lobes affected.

Complete blood count and reticulocyte count


Blood type and screen
Blood culture, sputum culture (if possible)
Viral studies and Mycoplasma titer (acute and follow-up)
Pulse oximetry
VQ scan
ECG

Chest X-ray showing bilateral lower zone consolidation.

Treatment:
Antibiotics:
Broad-spectrum intravenous antibiotic such a 2nd or 3rd-generation
cephalosporin as cefuroxime or IV Ceftriaxone (75 mg/kg/day) started
immediately plus an oral macrolide (erythromycin or azithromycin) to cover
atypical bacteria
Supplemental oxygen if hypoxemic (when the saturation is <90%).
Fluids with alkalinization:
Intravenous and oral fluids should be kept at maintenance.
Pain control:
nonsteroidal agents and narcotics Under no circumstance should opioid
administration be limited because of concern about preventing ACS; rather,
care must be taken to prevent ACS from occurring.
Transfusion:
Simple transfusion (10–15 cc/kg) – do not exceed post transfusion
Exchange transfusion – if
 no improvement with simple transfusion
 PaO2, <70 mmHg ,25% drop in baseline PaO2
 Acute congestive heart failure or acute right heart strain
 Rapidly progressive pneumonia
 Marked dyspnea with tachypnea
Adrenergic bronchodilators (to improve peak expiratory flow rates)
particular if history of reactive airways disease or if wheezing present
Steroids
may be beneficial for severe acute chest syndrome or if reactive airways
Incentive spirometry to reduce atelectasis
pain; the use of 10 breaths every 2 hr is an effective method to prevent ACS.
Mechanical ventilation as needed
Consider thoracentesis if significant pleural effusion
Prevention of ACS:
Patients with a history of recurrent ACS are candidates for preventative/
curative therapies including:
Prophylactic transfusions .
Optimal target HbS level is not known,but usually a goal of 30–50% is used
Hydroxyurea .
 Increase Hb F .
 starting dose of hydroxyurea is 15–20 mg/kg daily.
 increase in dose every 8 wk of 2.5–5.0 mg/kg
 up to a maximum dose of 35 mg/kg if no toxicities occur.
 It requires monitoring for side-effects (pancytopenia)monthly CBC

Stem cell transplantation .


The most severely affected children (1-5%) who do not respond to
hydroxyurea may be offered a bone marrow transplant.

This is the only cure for sickle cell disease but can only be safely carried
out if the child has an HLA-identical sibling who can donate their bone
marrow - the cure rate is 90% but there is a 5% risk of fatal transplant-
related complications.
Congestive heart failure
Management
ABC ,O2 100% , NPO, NG feeding
positive pressure ventilation may be required For patients with severe pulmonary edema

diuretics.
Lasix 1-2 mg/kg,
Spironolactone, 2-3 mg/kg/24 hr.

Digoxin (digitalization)
40mcg/kg give half the total digitalizing dose immediately and the succeeding two one-
quarter doses at 12 hr intervals later.

afterload-reducing agents and ace inhibitors.


Angiotensin-converting enzyme (ACE) captopril 0.3-6 mg/kg/24 hr /bid/tds
Afterload reducers are especially useful in children with
• heart failure secondary to cardiomyopathy
• in patients with severe mitral or aortic insufficiency.
• heart failure caused by left-to-right shunts.

α-and β-adrenergic agonists.


Dopamine 2-10 μg/kg/min if compromised kidney function + low cardiac output.
Dobutamine, 2-20 μg/kg/min.useful in treating low cardiac output
Epinephrine for patients with cardiogenic shock and low arterial blood pressure.

phosphodiesterase inhibitors.
Milrinone
initial loading dose of 50 μg/kg or intravenous infusion at 0.25-1 μg/kg/min
is useful in treating patients with low cardiac output who are refractory to therapy.
has both positive inotropic effects on the heart and significant peripheral vasodilator
effects and has generally been used as an adjunct to dopamine or dobutamine therapy

choice of drugs depend on condition of patient


if stable give oral digoxin and diuretic
if with pulmonary oedema need ABC + diuretics +/- digoxin
if with low cardiac output ,cardiomyopathy need ABC + ACI
if with low LT to RT shunt need ABC + diuretic +/- digoxin
if in shock
• cold cardiogenic shock with hypotension need ABC+ Epinephrine + Dopamine
• worm septic shock with hypotension need ABC+ norepinephrine + Dopamine
• cold or worm shock with NL BP need ABC+ Dobutamine + Milrinone
Respiratory Failure.
Types of Respiratory Failure.
Findings Causes Examples
Type I(peripheral) Ventilation/perfusion ARDS, atelectasis, pneumonia,
Oxygenation defect pulmonary embolus, BPD
defect Diffusion impairment Pulmonary edema, ARDS
interstitial pneumonia
Hypoxia
Decreased PaO2 Shunt Pulmonary AV malformation,
Normal PaCO2 congenital adenomatoid malformation
Rx O2
Type II (central) Hypoventilation head trauma, sedation
ventilation defect Neuromuscular disease (polio, GBS), ,
Hypoxia chest wall dysfunction (burns),
Hypercapnia kyphosis, severe reactive airways
Decreased PaO2
Increased PaCO2
Rx m.ventilation
Primary assessment: circulation, disability, and exposure
Circulation
The circulation assessed shock

To assess circulation, evaluate


 heart rate- tachycardia or bradycardia
 pulses (both peripheral and central)
 capillary refill time (>2 seconds) seen in dehydration, shock ,hypothermia
 skin color and temperature pallor ,mottling ,cyanosis
 blood pressure hypotension
 Other Signs Of shock including urine output < 1ml/kg /hr

Disability
The disability assessment is a quick evaluation of neurologic function.
evaluate disability by using the following:
• AVPU pediatric response scale
• response of pupils to light
• blood glucose test

Exposure
bleeding ,wound hypothermia, skin rash, petechiae or purpura
Assessment of circulation

Identification of Shock
After “evaluate,” the next step in the evaluate-identify-intervene sequence is to
identify if the problem is circulatory, respiratory, or both.
If the problem is circulatory, the child may be in shock.
Early identification and Rx of shock are key to improving outcomes in critically ill

Shock
Definitions
Defined as persistence hypotension state and poor perfusion regardless
volume expander and vasoactive agent

Types of Shock
Cold shock
When there increase systemic vascular resistance to improve venous return and cardiac
output and blood is diverted from non-vital areas like skin. Such children have cool
peripheries and are ‘shut down’. This is ‘ cold shock’ ,as hypovolemic and cardiogenic

Warm shock ’
When cytokines or neural responses cause vasodilatation and reduce SVR.
Such children will be tachycardic, but with inappropriately warm peripheries.
this warm shock — e.g. sepsis, anaphylaxis, and neurogenic shock
Identify Shock by type
Approach of Hypovolemic Shock
history of GE ? trauma?asthma, diabetes,NS, on steroids? antihypertensive, antidepressants
family history unexplained deaths or illnesses (CAH, IBEM)?
A irway clear or maintainable unless level of consciousness is impaired
B reathing Increased respiratory rate without increased effort (quiet tachypnea)
C irculation tachycardia ,weak pulses ,delayed CRT ,mottle skin, low UOP ,N/low BP
D isability changes in level of consciousness (irritability, agitation, anxiety or LOC)
E xposure extremities often cooler than trunk
Approach of Cardiogenic Shock
history of CHD? ,valvalopathy? chronic lung diseases?
A irway maintainable unless consciousness is impaired
B reathing tachypnea with Increased respiratory effort (pulmonary edema)
C irculation Signs of CCF (pulmonary edema,enlarged liver, distended neck veins),N/low BP
D isability Changes in level of consciousness (irritability, agitation, or anxiety, LOC)
E xposure : hypothermia ,extremities cooler than trunk , +/- Cyanosis
Approach to Distributive Shock (Septic/ Anaphylactic/ Neurogenic)
A irway typically clear or maintainable unless consciousness is impaired or anaphylactic
B reathing Increased respiratory rate without increased effort (quiet tachypnea)
C irculation Warm, flushed skin brisk capillary refill , bounding pulse .Normal, low, high BP
D isability Changes in level of consciousness (irritability, agitation, or anxiety, LOC)
E xposure : Fever or hypothermia , warm extremities + Hives ,petechial or purpuric rash
approach of Anaphylactic shock
• swelling of the face, lips, and tongue ,Hives ,Hypotension, Tachycardia ,wheezing
• history of New food today e.g. formula, dairy?
approach neurogenic shock are
 Hypotension with a low DBP, Normal HR or bradycardia
 history of spinal trauma?
approach of septic shock
• Fever or a low temperature , Changes in LOC (such as confusion or irritability)
• An elevated or decreased WBC or abnormalities in clotting ,Petechial or purpura
• history of chronic illness, malnutrition , immunodeficiency , immunosuppression Rx
Identify Shock by Severity (Effect on Blood Pressure)
The severity of shock is categorized by the effect on the child’s systolic blood pressure.
Compensated shock
 Signs of poor perfusion tachycardia,weak pulses ,delayed CRT,mottle skin,but normal BP.
Uncompensated - Hypotensive shock
 Signs of poor perfusion and low systolic blood pressure (hypotension)
Hypotension
- < 1 mon old age SBP < 60mm Hg
- 1 mon - 1yr old SBP < 70mm Hg
- 1 yr- 10yr old SBP < 70mm Hg +age in yrs x 2
- 10yr or older SBP < 90mm Hg
In children, the commonest forms of shock are: septic shock, i.e. distributive and
hypovolaemic shock secondary to trauma or gastroenteritis.
General management of shock consists of the following:
• Positioning of the child , Oxygen administration, Ventilation support
• Vascular access , investigation ,IV fluid therapy
• Monitoring , Frequent evaluation CRT, PR,RR,BP ,UOP chart , mental status and SaO2,ECG.
Position the child
Stable: allow to remain with caregiver
Unstable: if hypotensive, place child on her back unless breathing is compromised
Asses airway and breathing:
Maintain the airway, give high-flow O2 100% in all shock.
Be prepared to support ventilation with a bag-mask device as needed.
If a child look septic and has RD may need CPAP and EI.
If need Induction anaesthesia for EI , thiopentone cause vasodilatation, so using ketamine
Vascular Access: critical 2 line in hypotensive, preferred in compensated shock if cant, IO
Take blood for:FBC, glucose ,Ca , UEC, LFT, clotting ,BG ,blood cultures, ABG, chest xray
Fluid resuscitation
• Give bolus (20 mL/kg) of Isotonic crystalloid over 5 to 20 minutes for all shock.
• Can repeated upto 40 mL/kg and give 5 to 10 mL/kg over 20 min if poor heart function.
• Septic children may require over 100mL/kg of fluid, with risk of fluid overload.
• Blood loss is best replaced by blood O negative until cross-matched sample available.
• Correct hypoglycaemia and hypocalcaemia
Identify response to therapy As you give the fluid bolus, monitor :
• vital signs, Capillary refill time , Skin colour ,UOP and Level of consciousness
• Oxygen saturation with pulse oximetry and Blood glucose
If response once 40mL/kg of crystalloid has been given
CFT<2 sec , Nl pulses, HR,RR,BP ,UOP >1 ml/kg/hr, Nl mental status, Nl RBS and SaO2>70.
 fluid therapy should be titrated and ICU moniter
If no response once 40mL/kg has been given this Fluid refractory shock
Inform the duty anaesthetist and PICU consider intubation, and Access central line
If warm peripheries ,low BP, SaO2 > 70 , CVP< 10 cmHg ,this distributive warm shock
 If sepsis shock suspected- Fever, elevated or decreased WBC, purpuric rash
 Tittered volume with dopamine and noradrenaline
 give ceftriaxone or cefotaxime plus ampicillin if < 3mths old
 If anaphylaxis shock suspected- swelling of the face, lips, and tongue
 give adrenaline IM, hydrocortisone , antihistamine and ventoline nebulizer
 If neurogenic shock suspected – trauma, a low DBP , normal HR give noradrenaline
If cool peripheries, low BP and CVP< 10 cmHg, this cold hypovolemic shock
 Rule out hidden bleeding : give Blood even when not bleeding ,PRBC if hemolysis
 Rule out DIC : give FFP, platelets and/or cryoprecipitate ,give bicarb . for m.acidosis
 give hypertonic saline (3%) or colloids /albumin may be beneficial
 Rule out -CAH, hypoadrenalism , hypopititurism (risk for adrenal failure )
 draw blood for basal cortisol , do ACTH stimulation test
 give iv hydrocortisone2mg/kg bolus
 give dopamine in peripheral line and adrenaline by central line
If cool peripheries ,Nl BP and CVP> 10 cmHg this cold cardiogenic shock
 give duptamine and vasodilator PDE-milrinone or nitruprsside
if patient show worsening signs of RD, decrease in oxygen saturation
 Inform the duty anaesthetist consider intubation, if not done.
 Chest x ray ,echo , catheterization and see CVP
 Rule out non obstructive cardiogenic shock arrhythemiasis , duct dependent CHD
 DC shock if with tachyarrhythemia (VF, pulselesness VT or SVT)
 Adrenaline and atropine if bradyarrhytnmia
 PGE infusion if duct dependent CHD
 Rule out obstructive cardiogenic shock Co Ao , pneumothorax, P. effusion,P .embolus
 critical coarctation of the aorta needs to be repaired to improve.
 pulmonary embolus needs to be removed or undergo clot lysis
 Patients with symptomatic pericardial effusions or pneumothorax need immediate
removal of fluid or air, respectively. Before pericardiocentesis, arapid bolus of NS may
improve cardiac output. Nonetheless, the patient will not improve until obstructive
lesion is corrected.
 Rule out pulmonary oedema from fluid overload mainly with septic shock

If resistance shock see CVP


 If >15 consider diuretics and dialysis
 If 10-15 give inotrop
 If < 10 repeated isotonic salin

Dopamine or dubtamine dose 5–20 mcg/kg/min ,


The Rule of Six for drug infusion states that: 6 X body weight of patient this mg of a drug is
diluted in 100 ml,every ml/hr of the drug infused is equal to 1 mcg/kg/min,so give 5 -20 ml/hr

Adrenalin or noradrenalin dose 0,1–2 mcg/kg/min


The Rule of Six for drug infusion states that: 0,6 X body weight this mg of a drug is diluted in
100 ml, every ml/hr of the drug infused is equal to 0,1 mcg/kg/min, so give 1-20 ml/hr

milrinone dose loading 50 mcg/kg over 15 min.,then 1 mcg/kg/min


Congenital adrenal hyperplasia
Management adrenal salt-losing crisis
Plasma electrolytes must checked every few days for the first 4 weeks of life
detailed explanation with her parents, started on oral hydrocortisone replacement therapy.
Death can occur from adrenal crisis at the time of illness or injury.
The treatment of choice for adrenal crisis (any cause) is fluids, hydrocortisone, dextrose iv.
Iv fluid start with D5/NS 20m/kg then calculate deficiency + maintenance - bolus
Hydrocortisone is quick acting and at an emergency stress dose (100 mg/m2) saturates
all steroid receptors, causing a mineralocorticoid and glucocorticoid effect.
In addition, hypoglycemia is common during adrenal crisis, so the patient's glucose level
should be checked and an intravenous bolus of dextrose given if hypoglycemia is present.
2ml//kg of dext . 10%
The long-term management of both sexes is with long life cortison:
 glucocorticoids to suppress ACTH,(and hence testosterone) allow normal growth
 mineralocorticoids (fludrocortisone) if there is salt loss
 before weaning, infants may need added sodium chloride.
 additional hormone replacement to cover illness or surgery, as they are unable to
mount a cortisol response

Pt also need regular f.up and monitoring growth biochemistry,skeletal maturity(bone age )
and plasma androgens ,17- hydroxyprogesterone - insufficient hormone results in increased
ACTH secretion and androgen excess, which will cause rapid initial growth and skeletal
maturation at expense of final height; excessive hormonal replacement will result in skeletal
delay and slow growth
Affected females will require corrective surgery to their external genitalia, but as they
have a uterus and ovaries they should be reared as girls and are able to have children.
Surgery at 9 months of age to reduce clitoral size and separate the labia.
further genital surgery may needed to reduce clitoromegaly and a vaginoplasty before
sexual intercourse is attempted.
Psychological counselling and support were offered around puberty because females
often experience psychosexual problems, which may relate to the high androgen
Poisoning and intoxication

If pt has tachycardia ,hypertension, mydriasis,hypertherma ,hallucination ,this due to..


Stimulants = (Amphetamines,caffine,theophylin)+diaphorsis skin + arrhythmia + seizure
Hallucinogenic agent = (phencyclodine ,ectasy) + diaphorsis skin + seizure
Anticholenergic = (atropin ,TCA ,phenothiazine)+ dry flush skin,arrhythmia ,seizure , coma

If pt has bradycardia,hypotension,miosis,hypothermia seizure ,sediation ,coma this …


• opoid = (morphine ,heroin ,hydromorphone) + dry pale skin
• sediative /hypnotic = (benzodiazepine ,barbiturite ,alcohol) + dry pale skin
• cholinergic = (OPP ,inscectides) + diaphorsis skin
Drugs Associated with Major Modes of Presentation
Common toxic causes of cardiac arrhythmia
 stimulants: amphetamine ,cocaine ,theophylline
 anticholinergic : cyclic antidepressants ,antihistamines ,phenothiazine
 carbon monoxide ,chloral hydrate
 digitalis ,propranolol , antiarrhythmic : quinine, quinidine
 physostigmine
Causes of coma
 anticholinergic : cyclic antidepressants ,antihistamines ,phenothiazines
 narcotics/opioid
 sedating : alcohol ,barbiturates
 salicylates ,carbon monoxide ,lithium , methyldopa
Common agents causing seizures
 stimulants : amphetamine ,cocaine ,theophylline
 anticholinergic : cyclic antidepressants ,antihistamines ,phenothiazines
 hallucinogenic : phencyclidine
 cholinergic : organophosphate
 salicylates ,carbon monoxide ,lithium ,carbamazepine
Metabolic acidosis
 salicylates ,isoniazid, iron
 methanol ,ethanol, ethylene glycol
 paraldehyde, phenformin
hypoglycaemia
 salicylates ,ethanol ,isoniazid
 insulin ,oral hypoglycemic agents
 propranolol
hyperglycaemia
 salicylates ,isoniazid ,iron
 phenothiazine
 sympathomimetic
hypocalcaemia
 oxalate
 ethylene glycol
Radiopaque substance on KUB
 chloral hydrate, calcium carbonate
 heavy metals (lead, zinc, barium, arsenic, lithium, bismuth, as in pepto-bismol)
 iron , phenothiazine ,potassium chloride
 enteric-coated pills
 dental amalgam
Antidote
N-Acetylcysteine (Acetodote) Use in
 Acetaminophen poisoning
 150 mg/kg over 60min, then by 50 mg/kg over 4 hrs, then by 100 mg/kg over 16 hrs.
Atropine use in :
 Organophosphate poisoning and carbamate pesticides
 bradycardia due to atrioventricular conduction defects, β-blocking agents
 0.05 mg/kg repeated q5–10 min as needed; dilute in 1–2 mL of NS for ET instillation
Deferoxamine (Desferal) Use in
 Iron poisoning
 Infusion of 15 mg/kg/hr (max, 6 g/24 hr) IM:90 mg/kg/dose q8h (max, 6 g/24 hr)
Physostigmine (Antilirium) Use in
 Anticholinergic agents ,TCA
Pralidoxime (2-PAM, Protopam) Use in
 Organophosphate insecticides
Naloxone (Narcan) Use in
 Narcotics ,Clonidine (inconsistent response)
Octreotide Use in
 Sulfonylureas
Digoxin-specific Use in
 Digitalis glycosides poisoning (synthetic or natural)
 1 vial binds 0.6 mg of digitalis glycoside; ingested dose estimated from serum level
Diphenhydramine (Benadryl) Use in
 Extrapyramidal symptoms, acute dystonic reactions, allergic reactions
EDTA, calcium (calcium disodium, Versenate) Use in
 Lead, manganese, nickel, zinc, and perhaps chromium
Ethanol (ethyl alcohol) Use in
 Methanol, ethylene glycol
Flumazenil (Romazicon) Use in
 Benzodiazepines
Pyridoxine (Vitamin B6) Use in
 Isoniazid, Gyromitra mushrooms ,Ethylene glycol (investigational)
Fomepizole (4-methylpyrazole, Antizole) Use in
 Ethylene glycol, methanol
Glucagon Use in
 β Blockers, calcium channel blockers, hypoglycemic agents
Vitamin K Use in
 Coumarin
Methylene blue Use in
 Methemoglobinemia
Oxygen Use in
 Carbon monoxide
Paracetamol overdose
The acute toxic dose of acetaminophen is generally considered to be >200
mg/kg in children younger than 12 yr of age

Classic Stages in the Clinical Course of Acetaminophen Toxicity


STAGE TIME AFTER CHARACTERISTICS
INGESTION
I 1st 24 hr Anorexia, nausea, vomiting, malaise, pallor
II 24–72hr Resolution of earlier symptoms; RUQ pain
elevated bilirubin, PT, hepatic enzymes; oliguria
III 72–96 hr Peak liver function abnormalities
nausea, vomiting, and malaise may reappear
Hepatic failure ,coagulopathy, encephalopathy
IV 4 days–2 wk Resolution of hepatic dysfunction if survive

Treatment.
1- activated chacot 1–2 g/kg/ dose (max 100 g) and gastric wash (up to 6 hrs)
2- A minimum of 2 plasma acetaminophen concentrations should be obtained
the 1st at least 4 hr after the exposure and a 2nd sample 4–6 hr after the 1st.
and blot it in nomogram the Rumack-Matthew nomogram.
3- Give antidote N acetylcysteine-NAC therapy if falling within the toxic range ,
its effective when initiated early in the course of intoxication (within 8 hr),
but may have value even if started 24–36 hr after the ingestion
IV formulation, an initial IV loading dose of 150 mg/kg diluted with 200cc
dex 5% , infused over 15–60 min, followed by an initial maintenance dose
of 50 mg/kg diluted with 500cc dex 5% infused over 4 hr, followed by 100
mg/kg diluted with 1000cc dex 5% infused over 16 hr.
oral NAC dose (140 mg /kg loading dose then 70 /kg every 4 hr, for a total
of 17 doses), diluted to a 5% solution with soda or fruit juice to minimize
vomiting and give ondansetron ,continue antidote Rx until normalized LFT
4- . Liver function studies, including hepatic enzymes, bilirubin, and
prothrombin time, urea ,electrolytes ,RBS should be followed daily to
every other day in all patients with plasma acetaminophen concentrations
falling within the toxic range
Salicylates overdose.

The clinical presentation of acute poisoning differs from that of chronic toxicity.
Chronic toxicity results in signs and symptoms are easily attributed to other
causes, such as flu or other febrile illness. Young children are more susceptible
to toxic effects because they are less able to buffer the acid load produced by
salicylates. usually present with metabolic acidosis and hypoglycemia.
Hepatotoxicity occurs after chronic exposure.

Acute toxicity
If mild toxicity : nausea and vomiting ( gastric irritation) ,and tachypnoea.
if moderate toxicity Hyperglycemia with Hyperpyrexia stimulate respiratory
center, leading to hyperventilation, R. alkalosis with compensatory alkaluria
,high urine Hco3 ,K, Na ,when sufficient K has been lost by the kidneys, an
exchange of K for hydrogen ion occurs and urine acidify. “paradoxical
aciduria” in the continued R. alkalosis.
Dehydration >5–10% and progressive metabolic acidosis, caused by the
accumulation of lactic acid and other metabolic acids
if sever toxicity :
CNS changes as , Agitation, confusion seizure and coma (cerebral edema).
Pulmonary edema or hemorrhage may develop in more severe cases.
Hepatotoxicity may occurs after large acute ingestions.
Death results from pulmonary edema and respiratory failure, cerebral edema,
hemorrhage, severe electrolyte imbalance, or cardiovascular collapse.

Serial serum salicylate concentrations (4 hr post ingestion then every 3–4 hr)
Acute serum salicylate of >20 mg/dL should continued monitoring.
Bezoars tab, may be suspected if serum concentrations continue rise many hrs
serum concentrations of >70–100 mg/dL may produce life-threatening
effects.
Treatment.

1.gastric wash and decontamination(activated charcoal) , can repeated

2.Serial serum salicylate concentrations (6 hr post ingestion then every 4 hr)


Acute serum salicylate of >20 mg/dL should continued monitoring.
serum concentrations of >70–100 mg/dL may produce life-threatening effects.

3.Check CBC,ABG ,electrolytes ,RBS, LFT ,PT ,Urine pH measured hourly

4.aggressive rehydration ensure UOP >1ml/kg/hr so give 20ml/kg slain bolus


Then give D5 with 40mEq/l of kcl and 40mEq/l of bicarbonate (double DR)
to alkalized urine and increased Urinary salicylate elimination using “ion
trapping” increasing urine pH to convert a salicylate from un-ionized which is
reabsorbed in acidic urine to the ionized form which excreted in alkaline urine.
Each 1-unit increase in urine pH increases urinary salicylate clearance 4-fold.
The shift of salicylate to the ionized form also serves to decrease CNS
penetration because un-ionized drugs generally cross the blood-brain barrier.
Keep Urine pH at least 7.0–7.5, may be difficult to alkalize the urine without
adequately replenishing K stores because (bicarbonate and tahypnoea
exaggerated hypokalemia) so keep serum K at maximum upper limit = 5mmol

5.In severe cases of salicylate intoxication, Hemodialysis may be required


both to remove salicylate and to correct electrolyte abnormalities.
Indications for extracorporeal removal include serum salicylate
 concentrations of >100 mg/dL,
 sever refractory metabolic acidosis ,oliguria ,anurea (ARF)
 respiratory or cardiovascular instability (pulmonary oedema)
 changes in neurologic status (depressed consciousness or seizures)
Tricyclic antidepressants poisoning.
TCA such as amitriptyline(tryptizol) , imipramine(tofranil),doxepin.

The primary organ systems affected by TCAs are the CNS(most common) and CVS.
neuronal transmitter reuptake blockage in both the central and peripheral CNS
α-receptor blocking (Sedation)
anticholinergic (atropine) effects

leading to Drowsiness, lethargy, coma 30% ,resolves in 24hr


Seizures develop in 15% ,without warning, usually brief and resolve without Rx.
tachycardia (common) , dysrhythmias (VT,VF) , hypertension, hypotension(rare)
pt can developed hypoventilation which is poor prognosis
Other cardiac findings include
 slowing of myocardial conduction
 multifocal premature ventricular contractions
 ventricular tachycardia or fibrillation.
 widening QRS complex, QT prolonged
 T- flattening or inversion
 ST segment depression
 right bundle branch block
 complete heart block.

Anticholinergic syndrome, including mydriasis, disorientation, hallucinations,


hypertherma ,dry flush skin,urinary retention, and diminished bowel sounds.

Symptoms can develop as early as 30 min to 6 hr after ingestion ,reflecting the


anticholinergic properties of TCAs in slowing gastric emptying , motility.
TREATMENT.

1. general life support measures ABCD are instituted, and if indicated EI

2. Activated charcoal should be administered repeated 4 hr apart.


Emesis is contraindicated because of risk of aspiration (CNS depression)
Plasma TCA concentrations are not helpful in assessing the severity

3. The electrocardiogram (ECG) should be closely monitored for QT ,QRS


For prolonged QRS and hypotension give 20ml/kg NS and 2ml/kg IV NaHco3
repeated to achieve and maintain a serum pH of 7.45–7.50
Lidocaine and phenytoin is used to treat life threating dysrhythmias .
SVT no need treatment

4. Hypotension may not respond to fluid therapy, give norepinephrine.


Severe unresponsive hypotension is a poor prognostic sign.

5. Hypertension usually is transient and does not require treatment.

6. Seizures, if they require treatment, usually respond to iv valium.

7. Physostigmine, once promoted as an “antidote” for TCA toxicity,is a


dangerous agent that cause seizures and dysrhythmias , should not be used.

8. Extracorporeal removal is of no clinical value ,because of the large volumes


of distribution and the high degree of plasma protein binding of TCAs,

Asymptomatic children should be observed and the ECG monitored for at least
6 hr after exposure. If there any of the following manifestations of toxicity,
patient should be admitted for continued monitoring in PICU for 24 hr.
 QRS interval of >100 msec
 conduction defects
 altered mental status
 hypotension, or hypoventilation

Only completely asymptomatic children should be discharged after 6 hr


Organophosphate (Cholinesterase-Inhibiting) Insecticides poisoning
This household product, repeated low-grade exposure may result in sudden,
acute toxic reactions.This syndrome usually occurs after repeated household
spraying rather than agricultural exposure.
the clinical findings are the result of cholinesterase inhibition, which causes an
accumulation of acetylcholine.

Clinical presentation :
The onset of symptoms occurs within 12 hours of the exposure.
Muscarinic effect: meiosis, tearing, salivation , loss of reflexes and sphincter
control lead to vomiting, diarrhoea, urination also there bronchospasm
,sweeting ,tachy /bradycardia ,hypotension
Nicotinic effect: weakness, muscular twitching, respiratory failure and death
CNS effect: Dizziness, headache,Confusion, convulsions, and coma can occur
Hyperglycemia is common in severe poisonings

Treatment
1. general life support measures ABCD are instituted,catch vein
2. decontamination of skin, nails, hair, clothing with soapy water is important.
3. Activated charcoal should be administered repeated 4 hr apart.
4. Red cell cholinesterase levels should be measured as soon as possible.
If below 25% of normal indicates significant exposure.
5. Give an antidote
Atropine (anti muscarinic) plus a pralidoxime(cholinesterase reactivator) .
pralidoxime also ameolarate nicotinic effect so improve muscular weakness
starting dose of atropine is 0.01 mg/kg (0.5 -2mg)in a child ,repeated every few
minutes until airway secretions diminish(mydriasis not an stopping point)mus
continue until normalized gas exchange even if developed tachycardia.
Severe poisoning may require large quantities of atropine over 24 hours.
Because atropine antagonizes the muscarinic parasympathetic effects of the
OP but does not affect nicotinic receptor, not improve muscular weakness.
So Pralidoxime should also be given immediately in more severe cases , red
cell cholinesterase is less than 25% of normal, should be used in addition to—
not in place of—atropine ,in dose 25–50 mg/kg diluted to 5% dext, and
infused over 30 minutes at a rate of no more than 500 mg/min). repeated
after 1 hr then every 6–12 hours as needed ,most useful within 48 hrs
postexposure but effects 2–6 days.
Caustics ingestion or exposure.
May be alkali such drain cleaner, dish washer, or Acidic such battery fluid and
metal cleaner

Clinical presentation
• Oral or perioral burn ,retrosternal or abdominal pain
• Drooling ,vomiting ,stridor or DOB

Treatment.
1. thorough removal of the product from the skin by flushing with water.
Contaminated clothing should also be removed.
Ingested agents should be rinsed from the oral cavity.
Clean and irrigate eye with water or NS beside opthalamogist consultation
2. Emesis and lavage are contraindicated.
Activated charcoal should not be used because it does not bind these agents
and may predispose the patient to violent vomiting ,aspiration.
3. Patients should be evaluated for evidence of esophageal burns, and if
symptoms are present, oral fluids or solids should be withheld.
absence of visible oral injury does not preclude significant.
4. Chest x ray for pneumonitis ,medistinitis and aspiration
Abdominal x ray to exclude perforation(free air)
Endoscopy should be performed in symptomatic patients with perioral burn
,Drooling ,vomiting ,stridor or DOB risk of esophageal lesions
or those in whom injury is suspected on the basis of history.
5. The use of corticosteroids and esophageal stents is controversial.
6. Prophylactic antibiotics do not improve outcomes.
7. For pt with battery button ingestion there risk of alkali lekage soneed
gastroenterologists consultation and chest and abdominal x ray
Hydrocarbon inhalation/kerosene or gasolin

Supportive management is the mainstay of treatment

• need observation for 24 hrs, not give activated charcoal ,nor induced vomting

• Gastric lavage contraindicated because risk of aspiration

• AB are recommended only if associated with lower respiratory tract infection.

.
Iron poisoning
Five stages of intoxication may occur in iron poisoning:
(1) Hemorrhagic gastroenteritis, occurs 30–60 minutes lasts 4–6 hours.
(2) Phase of improvement, lasting 2–12 hours, which patient looks better.
(3) Delayed shock, which may occur 12–48 hours after ingestion.
Metabolic acidosis, fever, leukocytosis, and coma may also be present.
(4) Liver damage with hepatic failure ,hypoglycemia ,coagulopathy,coma,death.
(5) Residual pyloric stenosis, may develop about 4 weeks after the ingestion.
Once iron is absorbed from the GI tract, it is not normally eliminated in feces
but may be partially excreted in urine, giving it a red color prior to chelation.
A reddish discoloration of urine suggests a serum iron level > 350 mg/dL.
Treatment.
1. Because iron is radiopaque, an abdominal radiograph confirm the ingestion.
Repeat radiographs may help with assessment of the efficiency of gastric
decontamination methods. A negative result does not rule out iron ingestion
because only undissolved tablets can be seen
2. Serum iron concentrations should beat admission, 4 and 8 hrs after ingestion.
Serum iron concentrations of <500 mg/dL, indicate a low risk of toxicity.
Serum concentrations of >500 mg/dL indicate that significant toxicity is likely.
Serum iron concentrations are prognostic value in the asymptomatic patient
Also do CBC ,UEC,LFT,ABG ,blood group (Serum iron levels fall rapidly)
3. GI decontamination is based on clinical assessment.
Syrup of ipecac may be administered at home, with appropriate follow-up
Gastric lavage and whole bowel irrigation should considered in these patients.
4. Shock is treated in the usual manner.
5. Deferoxamine, a specific chelating agent for iron, is a useful adjunct in the
treatment of severe iron poisoning. It forms a soluble complex that excreted
in the urine. It is contraindicated in patients with renal failure unless dialysis .
IV deferoxamine chelation therapy should be instituted if:
--the patient is symptomatic
--the peak serum iron > 500 mg/dL (62.6 umol/L) at 4–5 hours after ingestion.
--Significant amount ingested from history
be given at a dosage of 15 mg/kg/h. up to 35 mg/kg/h if life-threatening.
Rapid administration cause hypotension, flushing, urticaria, tachycardia, shock.
Deferoxamine, 90 mg/kg IM every 8 hours (maximum, 1 g), may be given if IV
access cannot be established, but the procedure is painful, DFO stopped after
12–24 hours if the acidosis has resolved , the patient is improving ,urine clean.
Hemodialysis, peritoneal dialysis, or exchange transfusion can be used to
increase the excretion of the dialyzable complex.
Urine output should be monitored and urine sediment examined for evidence
of renal tubular damage.
Hypertension
If SBP and or DBP that is more than 90th percentile ,but < 95th centile ..this prehypertension
If SBP more than 95th percentile in clinic ,normal outside this white coat hypertension
Hypertension is defined as SBP and or DBP that is more than 95th percentile for gender
,age and height on three occasions , this stage I HT.
If SBP and or DBP > 5 mmHg more than 95th or > 99th percentile for gender ,age and
height on three occasions , this stage II HT /sever HT.
If SBP and or DBP > 5 mmHg more than 95th or > 99th percentile that is a marked
accelerated but not associated without life threating condition , this HT urgency.
If SBP and or DBP > 5 mmHg more than 95th or > 99th percentile associated with end organ
damage, this HT emergency.
In infants and younger children, systemic hypertension is uncommon, with a prevalence of
<1%, but when present, its usually indicative an underlying disease (secondary HT).
Common causes of pediatric hypertensive emergencies
Renal disease: (80%)
i. Nephritides – GN :
 Henoch-Schönlein purpura
 Postinfectious glomerulonephritis
 Systemic lupus nephritis
 Rapidly progressive glomerulonephritis
ii. Vascular:
 Hemolytic-uremic syndrome
 Renal artery ,vein stenosis and thrombosis
 Sickle cell nephropathy
iii. Congenital malformations:
 Polycystic kidney disease
 Tuberous sclerosis
 Hydronephrosis
 Renal hypoplasia
 Obstructive uropathy
iv. Miscellaneous:
 Renal failure with fluid overload
 Reflux nephropathy
 Renal tumors
Endocrine:
 Pheochromocytoma ,Congenital adrenal hyperplasia 11BHD
Cardiovascular:
 Aortic coarctation ,Subacute bacterial endocarditis
Drugs:
 Corticosteroids ,NSAIDs
 Oral contraceptive pills
 Theophylline ,Phenylephrine
Work up for hypertension

• If prehypertension review within 6 mon , no drugs ,increase activity ,decrease wt


• If hypertension in 3 reading ,Take full history ,examination and see BP in 4 limps :
• History include renal cardiac diseases , CNS symptomes and drugs history
• Examination include BP in 4 limps ,hydration state ,cardiac ,CNS deficit
• Investigation start with CBC , urinlysis ,electrolytes , creatinine ,urea
• if upper BP > lower BP = Co aorta do … C X ray , ECG and echo
• If equal BP ,high WBC,high creatinine = reflux nephropathy ,renal anomalies =US,DMSA
• If equal BP, prominent RBC = GN ,RVT,nephrocalcinosis = Ca , C3 ,ASO,ANA,biopsy
• If equal BP , normal urine ….. do further workup
• Plasma renine ,aldesteron
• Urine cortison and catacholamine
• TSH,T4 ,FSH ,doppler US ,CT angiography ,MRA ,angiography
• If all investigation normal = essential HT = Fasting RBS and lipid
• Consider refer to nephrologist
• Commonly used oral Antihypertensive Rx
• Captopril – 0,3 - 0,5 mg /kg /day /oral
• Lisinpril – 0,5 mg /day /oral
• Amlodipin – 0,05 - 0,5 mg /kg /day /oral
• Nifedin – 0,25 mg /kg /day /oral
• Lasix – 2 - 4 mg /kg /day /oral
hypertensive crisis
Hypertensive urgency: sever HT > 99th percentile without evidence of end organ damage
Hypertensive emergency: sever HT > 99th percentile with evidence of end organ damage
End organ damage includes:
- encephalopathy (papilledema) ,seizure , facial palsy ,hemiplegia
- blurred vision ,diplopia , retinopathy
- polyuria ,renal failure
- GIT bleeding
• If emergency hypertension , sever HT > 99th percentile + clinical signs end organ damage
• Admited to PICU ,cardiac and BP moniter
• IV acsess and Do CBC , urinlysis ,electrolytes , creatinine ,urea,C x ray ,ECG ,Fundoscopy
• quick history and physical examination
• Start IV anti HT
Labetalol /α,β blocker : onset : 2min , duration action : 2- 4 hrs
• 0.4 –1.0 mg/kg/h start ,then 1–3 mg/kg/h or 1mg /kg /bolus dose/even 10 min
• Mainly for AHT with high ICP (Asthma and overt CCF are relative contradiction)
Sodium nitroprusside/Vasodilator onset: sec -2min , duration action : 1-10 min
• 0.2–0.5 mcg/kg/min start , maintanance 3-5 mcg/kg/min(MAX 10 mcg/kg/min)
• Contraindicationin : Co aorta and AV shunt ,Monitor cyanide if >72 hr or in RF
Hydralazine / Vasodilator onset: 5 -20 min , duration action : 20-60 min
• 0.1–0.2 mg/kg/bouls dose only IV /4–6 hr ,Mainly if pulmnary odema
Nicardipine /CCB 1-3 mcg/kg/min IV infusion ( cause tachycardia)
Nifedipine /CCB 0.25–0.5 mg/kg/SL Repeat q 4–6 h
Diazoxide 2–10 mg/kg IV bolus
Furosemide 1–5 mg/kg IV mainly with volme overload
 Increase dose over next days
 Reducuion BP over days
Intravenous administration is often preferred so that the fall in blood pressure can be
carefully titrated. Because too rapid a reduction in blood pressure may interfere with
adequate organ perfusion, a stepwise reduction in pressure should be planned.
In general, the pressure should be reduced by about ⅓ of the total planned reduction
during the 1st 6 hr and the remaining amount over 72 hr.
 Faster reduction is recommending in pulmnary odema Start IV diuritcs If not respond to
Lasix need IE + MV + dialysis
• If hypertension urgency , sever HT > 99th percentile + no clinical signs end organ damage
 Admited to general word
 quick history and physical examination
 Do CBC ,electrolytes ,creatinine ,C x ray ,ECG
 Start oral anti HT
 captoprl 0.3- 0.5mg /kg/dose/every 6-12 hrs…if not respond added CCB
 amlodipen 0.1mg /kg/dose/every 12 hrs…if not respond added hydralizin
 hydralizin 0.7-1mg /kg/ day / every 6h-12 hrs
 indral 1-2 mg/kg/ day /every 6-12 hrs
 nifidipen 0.25 -0.5mg /kg/day/ every 12 hrs ….SL not recomended
 Increase dose over next days
 Reducuion BP over days not hours
STATUS EPILEPTICUS
Status epilepticus is continue clinical or electrical episodes seizure lasting at
least 30 minutes, or a series of seizures without complete recovery over the
same period of time.
important causes of status epilepticus include:-
 initial seizure (25–75%)(eg, anoxic or traumatic).
 cessation or disruption of AED, affects both groups (most common).
 systemic febrile illnesses (50%)
 intracranial infections (meningitis, encephalitis) (25%)
 poisoning
 acute metabolic disorders
 head injury

Status epilepticus most commonly in children age 5 years and younger (85%).
The most common age is 1 year or younger (37%), and the distribution is even
for each year thereafter (approximately 12% per year).

Classification for the causes of convulsive status epilepticus in childhood


1. Prolonged febrile seizure (25%)
2 .Acute symptomatic(25%)
 bacterial meningitis or viral CNS infection
 metabolic derangements or drug-related effects
 head injury ,hypoxia or anoxia and cerebrovascular disease.
3 .Remote symptomatic (25%)
 with a history of a pre-existing CNS abnormality more than 1 week.
4 .Acute on remote symptomatic
 febrile illness and occurred in a child with a CP or VPS
5. Idiopathic epilepsy related
 Convulsive status epilepticus occurred in children with idiopathic epilepsy
6. Cryptogenic epilepsy related
 Convulsive status epilepticus that occurred in children with cryptogenic
7. Unclassified
 Convulsive status epilepticus that cannot be classified to any other group
Treatment
1. ABCs
 Suction , oral airway ,NG tube ,lateral position.
 give O2 ,consider BVM ventilation and intubation may be necessary.
 Assess pulse, BP ,if hypotension support with IV fluids, Monitor vital signs.
2. Get IV/IO line;
 Check Dextrostix if low Give 50% glucose (1–2 mL/kg).
 Check serum glucose ,UEC ,Ca ,mg,Po4,LFT ,CBC,BUN ,ABG ,AED levels ,Toxicology
3 Begin stage I AED therapy; goal is to control status epilepticus in 20–60 min.
Rapid acting Benzodiazepines
IV/IO Lorazepam 0.05–0.2 mg/kg (4mg/ml) undiluted over 2 min. (maximum 4 mg)
 less effective with repeated doses
 longer-acting than diazepam (effect 6-12 hrs)
 Low lipid solubility , Action delayed 2 minutes
 Less respiratory depression than diazepam
IV/IO Diazepam 0.3–0.5 mg/kg undiluted over 2min (maximum 20 mg)
 may repeat 2nd dose in 5–20 min
 high lipid solubility ,very rapid onset and redistribution
 adverse effects are
 persistent hypotension
 respiratory depression
if no access vein
Intramuscular/ Buccal midazolam 0.2 mg/kg(15mg/3ml) .
Rectal diazepam 0.5 mg/kg rectal gel,
 typically reaches anticonvulsant levels within 5-10 minutes
 Intravenous solution given rectally is equally effective
If respond adjusted previous drugs , give maintenance Rx (Phenytoin or phenobarbital)
If SE persists go to stage II AED
Repeat benzodiazepines every 5-10 minutes IV /IO +
IV /IO Phenytoin 10–20 mg/kg IV (not IM) over 5–20 min; MAX 1000 mg;
 Onset 10-30 min
 monitor blood pressure , ECG because hypotension, dysrhythmia.
 Extravasation causes severe tissue injury
IV /IO Fosphenytoin 10–20 mg/kg IV or IM of "phenytoin equivalent."
• may be given more rapidly
• Onset 5-10 min
• extravasation well tolerated
Consider pyridoxine 100 mg/kg IV for child < 2 yrs
Give mannitol 20% 5 ml/kg over 20 min.
If SE persists after 10 min from given Phenytoin , give 3rd dose of diazepam

If respond adjusted previous drugs ,and continue maintenance Phenytoin 5mg/kg/d/bd


If SE persists after 5 min from end of Phenytoin go to stage III AED
 IV /IO Phenobarbital 5–20 mg/kg IV , Be ready for ventilation
If respond continue maintenance Phenobarbital 5mg/kg/d /bd

If SE persists after 10 min from end of infusion or 60 min from beginning go to stage IV
 EI ,ventilation,muscle relaxant – Propofol , if no access vein,Paraldehyde 4 ml/kg (PR)
 Midazolam infusion 0.2 mg/kg bolus then 5 mcg/kg/min
3 mg in 50 ml dext (1ml/hr=1mcg/kg/min/5ml/hr)

If not improve within 1-2 hr induced barbital coma


 Thiopenton 30mg/kg/hr (IV),till seizure stop then 5mg/kg/hr
Or
 Pentobarbital infusion 10 mg/kg every 1/2hr till seizure stop

4. Other approaches in refractory status:


 Monitor AED blood levels.
 Support respiration, blood pressure as necessary.
 Correct metabolic abnormalities (eg,low-sodium, acidosis).
 Consider underlying causes:
 Structural disorders or trauma. MRI or CT scan.
 Infection: Spinal tap, blood culture, antibiotics.
 Metabolic disorders: Consider lactic acid, uremia.
 medication levels
 toxin screen
 judicious fluid administration.
 Non - convulsive status epilepticus Consider EEG
Diabetic ketoacidosis

Classification of Diabetic Ketoacidosis


NORMAL MILD MODERATE SEVERE
HCO2 (mEq/L) 20–28 16–20 10–15 <10
pH (venous) 7.35–7.45 7.25–7.35 7.15–7.25 <7.15
Clinical No change Oriented, Kussmaul respirations; Kussmaul or depressed
alert but oriented but sleepy; respirations; sleepy to
fatigued arousable depressed sensorium to
coma

British Society for Paediatric Endocrinology and Diabetes (BSPED)


A. Emergency management
1. Resuscitation
2. Confirm diagnosis
3. Investigations
B. Full Clinical Assessment
1. Assessment of dehydration
2. Conscious level
3. Physical examination
4. Role of PICU
5. Observations to be carried out
C. Management
1. Fluids – volume , type ,oral fluids
2. Potassium
3. Insulin
4. Bicarbonate
5. Phosphate
C. Continuing management
D. Cerebral oedema
1. Features
2. Management
E. Other complications and associations

DKA there are:


• Hyperglycaemia: BG > 200 mg/dL
• acidosis : pH < 7.3 and Bicarbonate < 15 mmol/l
• dehydration : > 3%
• and/or deep rapid acidotic breath abdominal pain ,vomiting ,drowsy and coma
SUMMARY

TIME THERAPY COMMENTS


1st hour Emergency management
ABC O2 ,NPO , NG tube
100/kg IV bolus 0.9% NaCl volume expansion ; may repeated.
Full Clinical Assessment
 Assessment of dehydration
 Conscious level
 Physical examination
Confirm diagnosis/ Investigation Other investigations if indicated e.g.
• blood glucose  CXR,CSF, throat swab,
• urea and electrolytes  blood culture,
• ABG  urinalysis, culture and sensitivity etc.
• PCV and full blood count
Monitor
 CNS status.
 urine catheterization
 Use flow sheet
Insulin drip at 0.1iu/kg/hr Have mannitol bedside;
2nd hour until
Start after 1 hr 1 g/kg IV push for cerebral edema.
DKA resolution
50 units(0.5ml) insulin to 50 ml
0.9% saline in a syringe pump
1ml =0.1 units
Management / IV fluid Calculation IV fluid as
0.9% NaCl: + continue insulin deficit + 2 x maninantace – Bolus /48 hrs
Check = … ml / hr
 RBS hourly
 U & E’s /2 hours then 4 hourly
 Blood ketone every 1-2 hrs
20 mEq/L K If K < 3 mEq/L
• give 0.5 to 1.0 mEq/kg OR 80 mEq/L

Added 5% glucose
if RBS <250 mg/dL (14 mmol/L) if pH< 6.9 and shocked .
2nd hour until • give HCO3 2-4 mEq/kg
DKA resolution
After 12 hrs, if Na stable change
to 0.45% NS/D 5%/20 mmol KCl.
consideration single daily dose of
if ph > 7.3 • Fragmin heparine 100 units/kg/day
give Insulin drip at 0.05 iu/kg/hr

Variable Oral intake with SC insulin No emesis;


Stop Iv insulin after • HCO2 ≥ 16 mEq/L;
 1 hr if soluble insulin • normal electrolytes
 10 min if ultrashort insulin • Blood ketone < 1

You might also like